Этого треда уже нет.
Это копия, сохраненная 24 декабря 2017 года.

Скачать тред: только с превью, с превью и прикрепленными файлами.
Второй вариант может долго скачиваться. Файлы будут только в живых или недавно утонувших тредах. Подробнее

Если вам полезен архив М.Двача, пожертвуйте на оплату сервера.
Тред тупых вопросов №76 #352045 В конец треда | Веб
Тред вопросов о жизни, Вселенной и всем таком.

Спрашиваем то, за что в других местах выдают путёвку в биореактор. Здесь анонимные ученые мирового уровня критически рассмотрят любые гениальные идеи и нарисованные в Paint схемы.

Предыдущий тут: >>350370 (OP)
#2 #352062
>>351997
1 кг - 10-20 тысяч долларов и хорошо, если он не ебнет по дороге. ЯО и химию проще и дешевле в землю закапывать
#3 #352066
Проясните за гравитационные линзы. Насколько я понимаю, чем крупнее объект, тем больше света от соседних объектов он притягивает и поэтому благодаря одной галактике можно увидеть несколько других? Мне кажется, что я делаю огромную ошибку в своем предположении
sage #4 #352074
если фау-2 достигала высоты 180 км и потом стремительно неслась навстречу к Лондону то раскалялась ли её оболочка? Почему не пизданул заряд от нагрева от трения сквозь плотные слои атмосферы? Вай?
#5 #352087
>>352074

>раскалялась ли её оболочка?


Температура оболочки была около 700 градусов.

>Почему не пизданул заряд от нагрева от трения сквозь плотные слои атмосферы? Вай?


Бекоз оф амматол, которым набивали боеголовку, он довольно устойчив к вибрациям и сильному нагреву.
меркурий63 Кб, 618x526
#6 #352091
Есть ли на Меркурии глубина, на которой можно жить? Типа зоны обитаемости, где не холодно, не жарко, а в самый раз?
#7 #352092
у Плутона-Харона есть стабильные точки Лагранжа? как они вообще с барицентрами работают?
#8 #352093
>>352091
Перепады температур на Меркурии довольно существенны.
Я бы зарылся на полюсе, где всегда дубак, зато стабильный, и копал вниз, пока температурка не стала бы комфортной.
Насколько глубоко пришлось бы рыть - неизвестно. Геология Меркурия исследована чуть менее, чем никак.
#10 #352096
На каком расстоянии нейтронные звезды опасны для кораблей? Можно мимо нейтронки или пульсара пролететь на расстоянии 1/10/100 световых лет?
#11 #352097
>>352094
Крутая пикча.
Вот бы еще посмотреть как магнитные поля больших бинарных планет взаимодействуют, если такие есть вокруг других звезд. Они должны друг друга разогревать нехило, создавая любопытные структуры.
#12 #352101
>>352096

>На каком расстоянии нейтронные звезды опасны для кораблей?


Зависит от активности нейтронной звезды и ее типа.

>Можно мимо нейтронки или пульсара пролететь на расстоянии 1/10/100 световых лет?


Можно, главное ебалом в луч не совать, что бы брови не опалить.
#13 #352103
>>352101
а планеты вокруг нейтронных звезд светятся от радиации сами по себе?
#14 #352104
>>352103
Нет.
Тебе подробно нужно ознакомится с понятием радиолюминисценция и в каких случаях она возникает.
#15 #352105
>>352066
Фотоны от далёких галактик слишком разлетаются. Гравитационная линза чутка кучкует их и телескопы могут быть не такими огромными, ибо первой линзой выступает космический объект.

1. Может ли ракета летать на ОЧЕНЬ сжатом воздухе? Или всё равно будет недостаточно рабочего тела? Или так вообще нельзя?
2. Почему не делают таких стартовых столов, которые хотя бы чуть-чуть экономили бы топливо, подбрасывая ракету вверх?
3. Можно ли сделать конструкцию, в которой будет узкая и очень высокая труба, в которую будет загружаться ракета, а вокруг этой трубы будет ещё одна ШИРОКАЯ труба, в которую помещается какой-нибудь металлический или бетонный блин ровно под размеры трубы, потом узкая труба закрывается, блин подбрасывается ядерным взрывом, в широкую трубу подаётся воздух пока блин наверху, потом снизу узкой трубы открывается проход для воздуха и тяжёлый блин, падая, вытесняет воздух под собой в узкую трубу под огромным давлением и ракету очень быстро выпуливает вверх? Или, может, лучше этот блин поднимать каким-нибудь механизмом и потом просто сбрасывать? Предположим, что под всем этим будет большая пружина, компенсирующая падение многотонного блина и не дающая всему этому поломаться к херам.
#16 #352107
>>352066

>чем крупнее объект, тем больше света от соседних объектов он притягивает


"Свет", фотон безмассовая частица, дятел.
На частицу не имеющую массы, гравитация не действует от слова вообще. Нечего притягивать.

Чем "тяжелее" объект, тем сильнее он искривляет пространство. Формируя условную "линзу" большей площади и кривизны.
#17 #352109
>>352107
Ну что ты хочешь? Народ по-прежнему ньютоновской физикой мыслит. С создания ОТО всего лишь сотня лет прошла, ещё не до всех дошло.
#18 #352112
Если бы на Земле была бы не одна, а две - три - пять атмосфер давления, то как бы климат отличался? Был бы такой климат более комфортным или наоборот?

А если бы давление было 0.3... Ну предположим вместо азота много больше кислорода 0.5 - 0.7 атмосфер?

Что если океанической воды прибавить 15-25% а если убавить на столько же или лучше оставить как есть? Какой климатический результат будет предпочтительнее, а что относительно цивилизации?

Если одна из ниже перечисленных планет была в Солнечной системе вместо чертовых голых булыжников или ледышек ну есть кое что интересное в лице Титана, но он же зараза в миллиарде км и ещё и Криогенный в добавок, да ещё и спутник не самого малого по массе газового гиганта.... мда то полетели бы её прям ща в ближайшем времени гхм... адаптировать под себя:

1. Землеподобная, но суше чем наша, кислорода по меньше
2. Землеподобная, но влажнее с плотной атмосферой и жарче на пару десятков градусов глобальной температуры
3. Супер Земли, гравитация - 1.5 - 2.2 ЖЭ, на одной из них атмосферное давление на уровне океана опасненькое - 10 атмосфер
3. Нормальный Земля аналог но в атмосфере не пригодные для нормального дыхания газы
4. Земля сильно запачканная углеродом, то есть по всюду углеводороды и прочее, дышать сложно но много топлива и есть чем окислять
5. Углеродный мир с нормальной температурой, дышать нельзя, кислорода атмосферного для окисления нет.
6. Планета заваленная тяжёлыми элементами - ну в концентрации этак 15 раз большей чем на Земле, атмосфера не сильно приятнее Марсианской
7. Землеподобная с жирными ресурсами, кислородом для окисления и прочим, но горяченькая - 60-130 градусов по всей планете, плотная атмосфера.
Ну остальное не сильно приятнее имеющихся и так у нас голых камней.
8 Если бы вокруг Земли было бы вместо луны несколько высоко металлизированных планетоидов или же богатое металлами кольцо, добывали бы уже сейчас?
#19 #352113
>>352105

>ОЧЕНЬ сжатом воздухе


Нууу если ты нафигачишь несколько десятков тысяч атмосфер а лучше миллионов, массой в дофига тонн в место откуда будет газовать то... Только как ты заставишь балон не лопнуть делая его из тоненьких стенок алюминия \ углерода \ ещё чего там, потому что если делать толстенные то это всё будет весить столько, что НЕ ЭФФЕКТИВНО АШ ПИСЕЦ, так что как то так. А ещё чем собственно закачать воздух на ТАКОЙ уровень давления? Насосы то тоже не магические
#20 #352114
>>352105
2
3
Экономия будет писец копеешная, а сложность устройства и вероятность его поломки я ору какая, к тому же особо так эпично швырять космические аппараты не рекомендуются И УЖ ТЕМ БОЛЕЕ НЕ ЯДЕРНЫМ БЛЖАД ВЗРЫВОМ Хотя Орион планировали ядерными бабахами толкать, но ДОРОХА
tumblrohb5mpya5Y1remrfeo11280392 Кб, 663x1007
#21 #352118
Какого максимального размера может быть железокаменная планета?
Как Уран? Как Сатурн? Как Юпитер?
При каких условия столь объёмная планета будет обладать гравитацией как на Земле (до 150%)?
Можно ли на такой планете получить атмосферу, разряженную как на Марсе?
#22 #352120
>>352117
хуета уровня антисети
>>352118

> Какого максимального размера может быть железокаменная планета?


меньше двух диаметров

> При каких условия столь объёмная планета будет обладать гравитацией как на Земле (до 150%)?


ни при каких

> Можно ли на такой планете получить атмосферу, разряженную как на Марсе?


можно
150217214938804,8 Мб, 400x300
#23 #352122
>>352120

> меньше двух диаметров


Чому? Чому не трёх или семи?

> можно


Как?
Разве с ростом массы не начинает липнуть больше лёгких газов и, следовательно, атмосферы
#24 #352124
>>352113

> А ещё чем собственно закачать воздух на ТАКОЙ уровень давления? Насосы то тоже не магические


Взрывами.
#25 #352125
>>352124
Ракетостроение уровня \b\+++
#26 #352128
Доставьте картинку про упоротых быдланов с дачи, где они с голыми писюнами рассуждают о полётах на Глизе
#27 #352133
>>352122

>Чому не трёх


Трёх кстати можно, если насобираешь ТРИСТА(!) масс Земли из железокамней ОЧЕНЬ маловероятны такие условия, но если кидать больше её уже будет СЖИМАТЬ, накидаешь 100500 масс Юпитеров, получишь белый карлан, который постепенно сожмётся до диаметра Земли, если ещё кидать будет нейтроночка, если ещё то уже чдешка, если ещё то чдешка будет расти всё больше и больше, усё.

>>352122

>Как?


Ну не знаю жарить оче оче жёстким рентгеном миллиард, другой лет, сдирая атмосферку, или мистическая протопланетарная туманность из одной кучи металла и камней.
#28 #352140
>>352133
Можно же просто поставить планету близко к своей звезде. Как Венеру и ближе. Тогда звезда сама всё сдует, а останется какой-нибудь аргон и криптон, да серные окислы
#29 #352141
>>352140
Ты только что хтоническую планету.
https://ru.wikipedia.org/wiki/Хтоническая_планета
#30 #352142
>>352141
Вообще, я подумал об изначально каменной планете, но да, твой вариант круче.
#31 #352144
>>352142
Изначально массивная планета будет газовым гигантом, хочешь ты этого или нет.
И прекрати постить эротические половые гениталии, у нас детская борда!
#32 #352145
>>352144
Ну что поделать. если я порнопостер, и кроме робототехники, космоса и порнухи (и щепотку архитектуры и градостроения) меня ничего не интересует?
Могу постить просто эротику.
#33 #352176
Отчего мы дрочим на марс, а не на венеру?
И вобще где блджад мои современные программы по исследованию няшненькой?
#34 #352177
>>352045 (OP)
Правильно ли понимаю что все объекты в космосе находятся на чьей-либо орбите? Мб кроме черных дыр. На чьей орбите находятся Вояджеры?
#35 #352179
>>352176
Там адъ и израиль, неудобненько, микросхемы через 10 минут плавятся. То ли дело прохладный ламповый марс, по которому годами можно колесить.

>>352177
Вояджеры на галактической орбите.
В межгалактическом пространстве по идее притяжения настолько слабы, что об орбитах речи нет.
#36 #352182
>>352179

>Там адъ и израиль


Зато вокруг юпитера блядь курорт, да?
Нихуя, это не ответ.
#37 #352183
>>352182
Вокруг юпитера курорт. Если что никто не предлагает колесить его поверхность. А зонды туда сбрасывают после отработки по спутникам, ибо пускать зонд ради 5 минут горения в атмосфере это чересчур расточительно. В юпитер ли, в венеру - один хуй 5 минут проработает.
Поэтому если за те же деньги и дельту можно послать зонд примерно на то же расстояние но он будет исследовать 5 лет или 5 минут - выбор очевиден. Тебе нет?
#38 #352186
>>352183
С экономической точки зреня, он мне очевиден.
Но любопытсво блядь прижегает. Сука, имеем 3.5 говёных фоточек с поверхности, и успокоились.
#39 #352192
>>352186
Давай тогда не с экономической.
Что мы имеем и можем поиметь с венеры и с марса?
Планетология, вопросы образования СС, возможная колонизация.
Адъ и израиль на поверхности наверняка стерли с лица венеры всяческие геологически и планетологически значимые формации. На марсе же можно найти древние структуры несмотря на эрозию благодаря слабой атмосфере и отсутствию геологической активности.
Ну а про канализацию и объяснять не надо. На марсе в комбезе можно выжить. На утренней звезде для выхода наружу надо что-нибудь из аниме приправленное маняфантазиями, ирл никак не выйти.
#40 #352200
>>352192
Не факт. На Венере совсем другая химия, а следовательно совсем другая минералогия. Определенные минералы и в земной атмосфере живут не долго, окисляются и разрушаются. В атмосфере Венеры есть неземные минералы существующие только там, какие-нибудь соединения серы и углерода возможные лишь при высоких давлениях. Мы еще даже имен для них не знаем, но если они достаточно прочные, то могут существовать на глубине, в рассыпном виде. Геология венерианских недр тоже непонятна, может там мантия выплевывает лаву другой консистенции. Минералог во мне хотел бы все это пощупать, задокументировать, систематизировать, дать названия, разместить по полочкам с бирками.
Геологическую историю Венеры также можно восстановить если глубоко копать и во многих местах, сопоставить результаты. Мы сейчас не приспособлены к Венере, но это лишь недостаток наших материалов.
#41 #352201
>>352192
Да я совсем не форсю теже задачи для венеры что и марсианская экспедиция.
Ну зонд швырнуть раз в 20 лет же можно.
чтонибудь вроде изучения парниковых эффектов, ну или банально снимки пожирнее получить с орбиты, ветра там каталогизировать или ураганы найти, спускать ничего совсем не обязательно.
#42 #352204
>>352200

>Мы сейчас не приспособлены к Венере, но это лишь недостаток наших материалов.


Вот мы и подошли к корню проблемы.
Для сколь бы то ни было показательных результатов на планету надо заслать зонд, который смог бы в какую-нибудь спектроскопию, копания, а желательно вообще возврат.
Пока материалы позволяют послать только шарик с хладагентом и камерой который передаст фоточки и расплавится нахер. Охлаждаться там нереально, техника в любом случае нагреется до 500 градусов и выйдет из строя, т.к. чипов чтобы такое держали пока особо нет, если вообще есть.

>>352201
Вроде иной раз и поднимаются вопросы запуска зонда к Венере, но Марс все же более интересен в том плане, что там и грунт пощупать можно и зонд будет сидеть/кататься годами и наверняка выживет посадку. В случае с Венерой немалы шансы что он просто сгорит при посадке (что-то от температуры откажет).
Мы слишком мало летали на венеру потому, что не умеем туда летать. Мы не умеем туда летать потому, что слишком мало туда летали.
#43 #352208
>>352204
есть же вроде какие-то девайсы которые могут преобразовывать тепло в электричество? почему бы не охлаждать таким образом, перегоняя может не 100% но часть тепла в энергию?
#44 #352212
>>352208
У них низкий кпд.
#45 #352213
>>352208

>есть же вроде какие-то девайсы которые могут преобразовывать тепло в электричество


Таких не существует, это нарушает принципы термодинамики.

>почему бы не охлаждать таким образом, перегоняя может не 100% но часть тепла в энергию


Почитай про термодинамику. Хотя бы краткое изложение для умственно отсталых закончивших 3 класса церковно-приходской.

>>352212
Куда уж ниже, учитывая, что их не существует.
#46 #352214
>>352213
>>352208
Ну и сразу дополню, а то щас будут говном закидывать.
Есть термоэлектрические элементы, да.
Но они не тепло преобразуют, а градиент температур. Притом в определенном температурном диапазоне, в других работать не будут.
Так вот, когда твоя банка с аппаратурой нагреется до внешних 500 градусов - градиента не станет и работы не будет.

Эти же штуки могут создавать градиент температур от приложенного тока, таким образом можно немного (немного!) охладить внутренности, нагревая внешний корпус. Но эффективней в этом плане окажется компрессор с хладагентом расчитанным на высокие температуры.
#47 #352215
>>352213

>Таких не существует, это нарушает принципы термодинамики.


Чего не существует? Термоэлектрогенераторов? Или газовых холодильников?
#48 #352217
>>352215
Читай внимательней:

>тепло в электричество


Все эти штуки работают на градиенте температур. Просто так из тепла энергию не делают.
Прув ми ронг, буду рад.
#49 #352218
>>352213

>нарушает принцып термодинамики


Какой?
Энергия тепла системы переходит в другую, но никуда не девается. КПД низкий - энтропия всёровно рассеивает часть энергии в системе при конверсии.

ВСё это возможно весьма.
MW-EM488venusMG2016051110065850 Кб, 569x398
#50 #352219
А если сделать дирижабль, который летает в верхних слоях где прохладнее? и чтоб он летал всегда на ночной стороне. Думаю там вполне нормальные для техники температуры могут быть
#51 #352220
>>352218

>Энергия тепла системы переходит в другую


Откуда куда будет эта энергия идти когда у тебя зонд нагреется до температуры на поверхности?

>>352219
Думают о таком, пока даже о зондах-дирижаблях речи нет, а журналюхи уже город Беспин на венере рисуют.
#52 #352221
>>352219
Лучше сделать космический лифт, по которому крутить хладогент. Верхняя часть лифта рассеивает тепло в вакуум при помощи радиаторов.
Или же сделать гигантские зеркала, которые бы бросали на Венеру тени. Если сделать их довольно тонкими и надолго затенить, то можно снизить температуру планеты без всякого терраформинга.
#53 #352222
>>352221
Можно повысить температуру твоего очка моим хуем :3
worst post ever419 Кб, 803x688
#54 #352223
>>352221

>хладогент


>космический лифт


>на планете соразмерной земле по массе, когда на земле не смогли запилить


>на планете, где якорь оплавится, т.к. лучшие зонды дольше получаса не выдерживали


>на планете БЕЗ ГЕОСТАЦИОНАРНОЙ ОРБИТЫ


Обоссы меня Господь...
А зеркала - валидная идея, только это все равно мегаконструкция уровня минимум десятка МКС по усилиям и бюджету.
#55 #352225
>>352223

>ГЕОСТАЦИОНАРНОЙ


Афродиостационарной, если быть скрупулёзным.

>где якорь оплавится, т.к. лучшие зонды дольше получаса не выдерживали


Тут даже не в якоре дело, а в его постройке, зонды-строители оплавятся.
#56 #352226
>>352223
Можно еще так сначала изучаем атмосферу Венеры и особенно верхних слоев. Потом создаем материал с высокими отражающими свойствами и распыляем его в верхних слоях. Этот материал если долго будет оставаться в верхних слоях должен существенно снизить количество поглощаемого планетой тепла. Как на Земле такое говно происходит после извержения вулканов, но в сочетании с идеей, когда полярные шапки отражают свет. Так что пыль должна быть не черная, а белая, легкая и в атмосфере Венеры оставаться только наверху, не выпадая на поверхность.
#57 #352227
>>352223

> на планете БЕЗ ГЕОСТАЦИОНАРНОЙ ОРБИТЫ


поясните, плес
#58 #352228
>>352225

>Тут даже не в якоре дело, а в его постройке, зонды-строители оплавятся.


Строишь лифт в космосе с якорем и свистоперделками
@
опускаешь на орлах шарах
#59 #352229
>>352227
Оче медленно вращается. Сиречь ГСО будет лежать за пределами сферы влияния планеты.
#60 #352230
>>352223
йО, а если лазером пробить атмосферу Венеры и в окно вставить лифт?
#61 #352231
>>352226
Такого не было еще. Речь обычно идёт всего лишь о колоссальных размеров зеркале (точнее системы зеркал для стабилизации, т.к. основное как парус улетело бы нафиг из солнечной системы)

>>352228
Да пофиг, афродиостационарной орбиты нет же.

>>352230
Ты только сделаешь мой член твёрже разогреешь и без того горячую атмосферу.

Я понимаю, тут тред тупых вопросов, но не настолько же.
#62 #352232
>>352230
Пробил хуем твою щеку.
image698 Кб, 640x640
#63 #352234
>>352226
в америке в водохранилище чтоб вода не цвела засыпали шариков. то есть если мы засыпим зеркальные шарики малой плотности, и если их будет дохуя то они закроют отражающей оболочкой планету
#64 #352235
АЛЛО, во что вы спейсач превратили? Мамаебов милости прошу к нашему шалашу >>285318 (OP)
#65 #352236
>>352234
Вот и я о том же. Осталось только найти как сделать столько шариков/пыли. охладив планету нужно решать что делать дальше. Вдруг атмосфера отчебучит фокус какой-нибудь непредсказуемый и все они выпадут на поверхность.
#66 #352237
>>352231
а что если это моя цель? Вдруг я хочу нагреть Венерку еще сильнее, чтобы испарить атмосферу совсем? Тогда зеркала можно сделать фокусирующими и нагревать на полюсах(или экваторе?), чтобы молекулы газа их оттуда легко уносило.
#67 #352238
>>352236
Хуйня эта затея, на поверхность планеты будет то же количество солнечной энергии приходить.
Надо зеркало ДО планеты вешать, тогда она станет остывать. Многие столетия.
#68 #352239
>>352238

> на поверхность планеты будет то же количество солнечной энергии приходить.



нихуя
https://ru.wikipedia.org/wiki/Похолодание_535—536_годов

Необязательно шарики или пыль распылять. Мне вот пиздецки импонируют нити в пару миллиметров. Что-нибудь типа асбеста распылить в атмосфере Венеры и пизда жаре.
#69 #352240
>>352239
Я слегка неверно выразился.
Атмосферный способ тоже сработает, но это чуть ли не большего масштаба постройка получается - такую-то массу завезти и распылить в атмосфере и чтобы изменения в атмосфере не заставили эту хуиту выпасть осадками и расплавиться, и чтобы эта хуита покрывала более-менее равномерно, а не кучковалась итд.
В итоге проще и надёжней тупо закрыть часть (а лучше всю) планеты от солнца.
#70 #352241
>>352240
Интересно как долго она будет остывать. Серная кислота снегом выпадет на поверхность со временем? А если открыть небольшие окна в зеркале и пустить свет, то у нас будут реки из растопленной серной кислоты? Ебанутый цикл дня и ночи все равно будут оставаться, что бы ты не делал с атмосферой.
#71 #352242
>>352241

>Ебанутый цикл


Поэтому солетта не одна, а группировка. Они не только поддерживают друг друга в квазистабильных поизциях, но и могут попеременно освещать разные полушария планеты.
Но это маняфантазии и при нашей жизни такого не будет потому, что нахуй Венеру, все на Марс!
#72 #352248
>>352177

>Мб кроме черных дыр


Почему? Ты понимаешь чёрную дыру как дырку? Думаешь, она не может крутиться вокруг, например, центра галактики?
Ну и само понятие орбиты придумано человеком. Это просто следствие того, что из-за воздействия гравитации путь движения тел может закругляться в овал.
#73 #352250
Космология это раздел астрономии или наоборот?
#74 #352251
>>352250
Космономия это раздел астрологии. Пестросян.джпг.
А вообще - да.

>КОСМОЛОГИЯ – раздел астрономии и астрофизики

#75 #352252
>>352250
Первое.
amazonka842 Кб, 750x494
#76 #352262
Представим что есть две планеты - на одной гравитация больше земной, а другой меньше земной. На которой из них будут более полноводные, глубокие и мощные реки?
#77 #352263
>>352262
На той, где больше воды.
#78 #352264
>>352262
На той, что в зоне златовласки и у которой много воды. А там уже как ландшафт покажет.
У какой-то планеты такие-то горные гряды будут, и реки размерами с моря через полушария течь, а какая-то планета из-за своей тектоники просто набор островов и речки-говнотечки.
Для одинаковых внешне планет, полагаю, реки глубже будут у той, что массивнее, т.к. эрозия сильнее.
#79 #352265
>>352262
>>352263
>>352264

>На которой из них будут более полноводные, глубокие и мощные реки?
Где осадков выпадает больше, ёпта.
15041859602380 (1)721 Кб, 1920x1200
#80 #352274
>>352045 (OP)
Если черная дыра может растягивать пространство, то почему галактика не может? Быть может галактики вовсе не разлетаются по вселенной от большого взрыва, а растягивают пространство своим весом?
#81 #352276
>>352274
Любая масса "растягивает пространство". Вообще любая. Даже твоя мамаша.
Проблемка есть в том, что даже не масса (видимая), видимо (простите за каламбур), тоже в этом участвует.
А так - да. Галактики искажают ПВК.
И нет, не растягивают. Их удаление связано с расширением именно Вселенной.
#82 #352282
>>352045 (OP)
ОЧЕНЬ ВАЖНЫЙ ВОПРОС ЗАСЛУЖИВАЮЩИЙ ОТДЕЛОГО ТРЕДА: почему звёзды рождаются разными?
#83 #352283
>>352282

>ЗАСЛУЖИВАЮЩИЙ ОТДЕЛОГО ТРЕДА


Нет. Потому, что ответ в одно слово:
масса
Разная масса = разные звёзды.
#84 #352284
>>352283
дай ссылку на поподрбнее. можно на англ

тут все студенты физфаков?
#85 #352285
>>352284
Начни с этого:
https://ru.wikipedia.org/wiki/Диаграмма_Герцшпрунга_—_Рассела
https://ru.wikipedia.org/wiki/Звёздная_эволюция
Возвращайся, если что-то непонятно.

>тут все студенты физфаков?


Нет. Мне 29, например.
#86 #352287
Ананасики
вопрос какую роль играют фотоны в строении атома, не берутся же они из неоткуда когда атом слегка даже нагреть
#87 #352288
и второй вопрос, что конкретно мы видим на фотографиях атомов, очевидно что протон, нейтрон и электрон слишком маленькие и находятся в очень малых количествах в атоме
однако некое конкретное массивное тело из чего-то плотненького таки состоит?
#88 #352289
Если как пишут оболочка атома состоит из электронов, то при ионизации атома его нельзя будет увидеть в микроскоп? электроны же уедут?
#89 #352290
Есть главная последовательность звёзд.
Но ведь рождаются и фиксируются звёзды выходящие за пределы той кривой. Почему? Как часто? Чем определяются?
Скоро ли мы найдём голубые карлики или бурые сверхгиганты?
#90 #352293
>>352288

>на фотографиях


не существует. есть лишь семтаические изображения.
#91 #352295
>>352287

>не берутся же они из неоткуда когда атом слегка даже нагреть


У тебя ответ в вопросе. Как ты нагреешь атом без фотонов?
#92 #352297
>>352290

>Но ведь рождаются и фиксируются звёзды выходящие за пределы той кривой.


Выходящие за пределы звезды не рождаются, а становятся, пройдя эволюционный путь.

>Скоро ли мы найдём голубые карлики или бурые сверхгиганты?


Никогда. В этом вся суть звезд. Их спектр определяется массой.
#93 #352302
Если бы Земля вращалась в 5 раз быстрее, то на экваторе гравитация ощущалась бы слабее? Как быстрое вращение влияет на недра больших планет? Ядро тоже сплющивается, или оно круглое и у него другая скорость?
#94 #352305
>>352295

>Как ты нагреешь атом без фотонов?


Столкновением с виртуальными частицами.
#95 #352306
>>352302

>Если бы Земля вращалась в 5 раз быстрее


В эпоху первых бактерий, когда луна была оче близко, сутки длились 4 часа. Все норм было.
#96 #352307
>>352306
А сейчас вращение увеличивает или замедляется?
#97 #352308
>>352305
Лолблядь.
#98 #352309
>>352307
Замедляется. Луна тормозит Землю.
#99 #352311
>>352289
Атомы итак не "видят" в электронный микроскоп. Их "нащупывают" излучением. Длины волн видимого спектра банально превышают размеры атомов в любом случае. детектятсЯ они не столько поиском в пространстве, сколько откликом на эм-воздействия. Ничто не мешает нащупывать голые ядра.
#100 #352312
>>352311
Вообще-то мешает. И ядра отдельно от атомов только совсем недавно научились "фотографировать".
#101 #352320
Если бы у нас был мощный лазер на орбите Земли, мы смогли бы направить его на Пионер или Вояджеры и придать им небольшое ускорение? Если бы мы могли управлять их солнечными панелями, мы смогли бы чуть чуть управлять вектором ускорения?

Интересуюсь потому что в твердой sci-fi часто говорят про лимит топлива на космических торпедах, который не позволяет им бесконечно разгоняться и маневрировать. А если торпеду лазером разгонять и помогать ей маневрировать после исчерпания лимита топлива?
#102 #352321
Могут ли существовать планеты с двумя ядрами?
#103 #352322
>>352320

>Пионеры


>Вояджеры


>Солнечные панели


Хорошо 1 сентября отметил?
#104 #352323
>>352321
Нет.
#105 #352324
>>352322
Да, на линейке постояли, послушали про нашу страну. А теперь на вопрос отвечай - из намека я понял, что солнечных батарей там нет. Но хоть антенну крутить можно?
sage #106 #352325
Я пролетаю возле нейтронной звезды. Совсем рядом. И пытаюсь посмотреть на неё в телескоп или с помощью технических способов. Как будет выглядеть поверхность звезды? Давайте откинем вещество которое падает на неё и светится. Именно как выглядит поверхность в видимом диапазоне? Будет ли на неё зайчик от мощного фонарика? Или это чёрная-чёрная поверхность так как это ядерный материал и не способный излучать-переизлучать фотоны и отражать солнечный свет.
#107 #352326
>>352320
Можно, но нужно рассчитать мощность лазера. И ускорение будет крайне малым, без паруса это не эффективно. Слишком мала площадь аппарата. А вот существующий курс можно скорректировать, собсна на вики есть статья "Эффект Пионера".
#108 #352327
>>352325
Абсолютно гладкий шар. Прям совсем гладкий. Если быстро вращается, то немного сплюснутый. Если горячий - светится ослепительно. Все зависит от типа НЗ и насколько она горячая. У нейтронных звёзд сравнительно небольшой массы может быть кора из железа. Цвет железа - понятно какой. Но из-за гравитации длинна волны фотона будет растягиваться. Так что зависит от того, чем на неё светить.
#109 #352328
Можно создать такую мощную черную дыру, чтобы она остановила расширение Вселенной хотя бы локально? И если создать некую форму с такими дырами в её углах, то получится ли продлить нашу связь с другими галактиками?
#110 #352330
Посоны, хочу в качестве курсача по матану какую-нибудь практическую задачу по орбитальной механике или другому йобакосмосу. Что посоветуете?
3010a2bfb7495fb6f558a70032425ef3a90ad355f4a3fd7b0c972c59790[...]14 Кб, 724x596
sage #111 #352331
>>352326
Можно еще вопрос? Я знаю, что в космосе невозможен стелс? Но можно ли обмануть сенсоры, замаскировать массу объекта в космосе или же создать ложное впечатление у наблюдателя? Допустим к Солнечной системе летит флот кораблей, он может при помощи роя фальшивых дронов обмануть наземные телескопы, внушив им, что корабли обладают реальной массой? Можно же и выхлоп мощных двигателей сымитировать на дронах, введя систему обороны в заблуждение? Допустим флот наступления разделяется и пытается пробить оборону с трех направлений. Оборона должна будет направить на перехват каждого из флотов свои средства. Но если они распылят средства на ложные приманки, то на гигантских расстояниях цена их ошибки должна быть роковой?
sage #112 #352332
>>352330
Вычисли эффект Ярковского для гипотетического астероида со спутником.
Посчитай время, которое потребуется на выпадение колец на Харикло.
Тред почитай этот и предыдущий, мы тут кучу всякой херни обсудили, но реальных цифр и вычислений не проводили.
sage #113 #352334
>>352327
то есть почти зеркальная при маленьких размерах и абсолютно чёная при значительных?
sage #114 #352335
>>352334

>при маленьких размерах


>при значительных?


Нет нейтронных звезд маленьких и значительных. Между пределом Чандрасекара и пределом Оппенгеймера-Волкова разница не большая. 1.45-2.65 Mʘ примерно. Так что радиус НЗ не слишком варьируется.

>то есть почти зеркальная при маленьких размерах и абсолютно чёная при значительных


Так сказать нельзя. Мало информации. Нужны все параметры, и масса, и температура, включая то, чем светить на звезду, так как от длины волны, которая отразится можно будет судить будет ли что-либо видно, или нет.
#115 #352336
>>352331

>Можно же и выхлоп мощных двигателей


Вообще лол. Сразу скажут "ну какой идиот будет в космосе так дымить? Явно приманка" и хер забьют. А реальный боевой флот не нужен включит двигатели только перед самым подлётом к цели чтобы затормозить, им незачем весь полёт это делать.
Да и почему стелс невозможен? Если у тебя есть деньги построить такой флот, то будут и деньги на несколько больших линз, проецирующих свет от звёзд позади. Хотя и это, по сути, ненужно, просто херачишь перед флотом большой чёрный квадрат и ставишь фонарики туда, где должны быть звёзды. Электронику по возможности вырубаешь. На подлёте включаешь двигатели вместе с остальной аппаратурой и тут же ингейдж. Только я не знаю зачем это нужно если можно просто издали просканировать все защитные системы и послать в каждую по боеголовке, сбросить ещё парочку на саму планету и потом прилететь парой корабликов чисто для того, чтобы победу объявить и правительство назначить.

>то на гигантских расстояниях цена их ошибки должна быть роковой?


Это на орбите земляшки-то гигантские расстояния? Все атаки, которые можно произвести с дальней орбиты, можно произвести и с границы солнечной системы, а на ближней орбите там расстояния уже никакие, преимущество максимум в пару минут будет.
#116 #352337
>>352328
Нельзя. Массу где возьмешь? Родишь? Даже если ты несколько ближайших галактик полностью конвертируешь в одну ЧД их суммарная гравитация не станет сильнее ни на граммулик.

>хотя бы локально?


А ты не в курсе что кластеры из галактик и так удерживают друг друга гравитацией и что разбегаются только скопления галактик?
e14455bc3b85789798d50406f1cf029c--warhammer-fantasy-warhamm[...]43 Кб, 706x519
#117 #352338
>>352336

>просто херачишь перед флотом большой чёрный квадрат и ставишь фонарики туда, где должны быть звёзды.



Маскировка уровня пикрелейтед.
Ибецилом для имбецилов придуманная.
#118 #352339
>>352338
Тебе чего нужно? От визуальных наблюдений с телескопов закрыться? Ну и чем плох кусок чёрной материи, не отражающий свет? На данный момент телескопы не смогут засечь даже какой-нибудь сраный метеорит, двигающийся из-за пределов СС на Землю. Так что эта маскировка так, на всякий случай. Фонарики, само собой, с отрегулированной мощностью и вообще полностью имитирующие свет имитируемой звезды. От радаров — я уже сказал, технику выключить, плюс можно позади того же чёрного квадратика поставить слой чего-нибудь такого, что будет помогать флоту не палиться.
#119 #352340
>>352339

>космические флоты


>станции обнаружения из оптических телескопов малой апертуры, с покрытием меньше 1%


>Закрываться тряпочкой.


>Фонарик светит, как звезда.



Ты ещё тупее, чем казался на первый взгляд.
#120 #352341
>>352336

>Это на орбите земляшки-то гигантские расстояния?


так я на картинке изобразил орбиту Плутона, но забыл сказать, что речь идет о конфликте довольно продвинутых противников. У Земли как бы тоже должны быть флота и желательно задержать флот неприятеля навязав ему бой на самых дальних подходах, где-то за орбитой Марса минимум. Но нужно выйти на перехват реального флота, а не разогнаться наперехват ловушке, которая лишь имитирует тепловые и отражательные сигнатуры флота.

>чёрный квадрат и ставишь фонарики туда, где должны быть звёзды.


Паралакс нельзя измерить?

>если можно просто издали просканировать все защитные системы и послать в каждую по боеголовке


Если человечество достаточно разовьется, то защитные системы, радиолокационные станции, телескопы могут быть на тысячах астероидов нашей Солнечной системы летающих по самым причудливым орбитам и наблюдающих во все стороны. Если допустить, что данные со всех этих ресурсов будут стекаться в стратегический суперкомпьютер, то он смог бы составлять приблизительную картину системного поля боя. Из-за количества станций, спутников и радаров на разных орбитах вывести их все из строя будет нереально, но их все равно можно обмануть. Наверное.
#121 #352342
Какую максимальную скорость можно разогнать космическому кораблю, запущенному с Земли, используя гравитационные маневры (Солнце+планеты), не важно сколько его будут разгонять (года-десятилетия)? Можно ли ее будет поддерживать, если направить наш самокат к ближайшей системе (5-50св.лет). Сможем ли мы держать связь с нашей йобой?
131880065784364 Кб, 450x312
#122 #352343
>>352242
Еще идея - гигантские зеркала можно использовать для защиты Земли от гамма-вспышек. Думаю никто не возражал бы, если бы они висели далеко от Земли, но закрывали планету с определенных опасных направлений.

Также они в теории могли бы защитить от межзвездных околосветовых kinetic kill vehicles. В теории.
#123 #352345
>>352332

> Вычисли эффект Ярковского для гипотетического астероида со спутником.


есть же астероид_на_дому, зачем такое считать?
#124 #352348
>>352340
Что не так? Я же говорю, на всякий случай. У меня ощущение, что ты тролль обычный, какие блжад проблемы в том чтобы выпустить из специализированного фонарика тонкий луч света, поправляя его мощность и всё остальное на подлёте чтобы было похоже на свет звезды? Если уж так докапываешься то объясняй, пидарас, уже который пост от тебя только задорнов_ну_тупые.jpg и ничего кроме можно заранее выбрать область без ярких звёзд и подлетать с её стороны, тогда можно просто тряпку перед флотом вести или же каждый корабль спереди неотражающей материей обить.
>>352341

>Паралакс нельзя измерить?


Так а кто додумается мизерный кусочек неба как-то там сканировать и вообще внимание на него обращать? Ты понимаешь насколько мал флот на фоне громадной солнечной системы, уж не говоря про остальной космос? Ну а если у тебя читерские радары, то я уже сказал: двигателями кораблям незачем дымить, нападающий флот берут и сразу пуляют в Землю по чёткой траектории ещё на подлёте к системе.

>телескопы могут быть на тысячах астероидов нашей Солнечной системы


Ну если мы станем поехавшей империей-завоевателями, то да.
Я не понимаю, чего ты хочешь. С такими технологиями радары смогут сразу 3D-модель вражеского флота составить. Какие приманки, какое двигателями подымить? Если там будут стандартные радары, то я уже сказал, 30 кораблей могут не светиться если просто обить их неотражающей материей и выключить всё оборудование, если там прям звиздец радары везде-везде, то можно сделать 90 кораблей, но 60 из них будут без никакой начинки и некоторые даже без двигателей, а 30 будут боевыми и оснащёнными. Тогда не вижу проблемы сделать их приманкой. Но если говорить реалистично, то к тому времени технологии разовьются уже настолько, что будут распознавать и такие приманки. И само собой тебе тут никто не скажет как замаскировать две приманки под два флота именно из-за того, что никто даже представить себе не может, какие именно к тому времени выдумают уловки для распознавания приманок. Всё, что мы тут навыдумываем, к тому времени уже 100500 раз как законтрят и выкинут потому что эти идеи устареют.

>Можно ли ее будет поддерживать


В космос никто кисель не проливал, поддерживать её не требуется. Лишь молиться чтобы в корабль не врезалась мимопылинка, ну или выдумывать защитные экраны.
#124 #352348
>>352340
Что не так? Я же говорю, на всякий случай. У меня ощущение, что ты тролль обычный, какие блжад проблемы в том чтобы выпустить из специализированного фонарика тонкий луч света, поправляя его мощность и всё остальное на подлёте чтобы было похоже на свет звезды? Если уж так докапываешься то объясняй, пидарас, уже который пост от тебя только задорнов_ну_тупые.jpg и ничего кроме можно заранее выбрать область без ярких звёзд и подлетать с её стороны, тогда можно просто тряпку перед флотом вести или же каждый корабль спереди неотражающей материей обить.
>>352341

>Паралакс нельзя измерить?


Так а кто додумается мизерный кусочек неба как-то там сканировать и вообще внимание на него обращать? Ты понимаешь насколько мал флот на фоне громадной солнечной системы, уж не говоря про остальной космос? Ну а если у тебя читерские радары, то я уже сказал: двигателями кораблям незачем дымить, нападающий флот берут и сразу пуляют в Землю по чёткой траектории ещё на подлёте к системе.

>телескопы могут быть на тысячах астероидов нашей Солнечной системы


Ну если мы станем поехавшей империей-завоевателями, то да.
Я не понимаю, чего ты хочешь. С такими технологиями радары смогут сразу 3D-модель вражеского флота составить. Какие приманки, какое двигателями подымить? Если там будут стандартные радары, то я уже сказал, 30 кораблей могут не светиться если просто обить их неотражающей материей и выключить всё оборудование, если там прям звиздец радары везде-везде, то можно сделать 90 кораблей, но 60 из них будут без никакой начинки и некоторые даже без двигателей, а 30 будут боевыми и оснащёнными. Тогда не вижу проблемы сделать их приманкой. Но если говорить реалистично, то к тому времени технологии разовьются уже настолько, что будут распознавать и такие приманки. И само собой тебе тут никто не скажет как замаскировать две приманки под два флота именно из-за того, что никто даже представить себе не может, какие именно к тому времени выдумают уловки для распознавания приманок. Всё, что мы тут навыдумываем, к тому времени уже 100500 раз как законтрят и выкинут потому что эти идеи устареют.

>Можно ли ее будет поддерживать


В космос никто кисель не проливал, поддерживать её не требуется. Лишь молиться чтобы в корабль не врезалась мимопылинка, ну или выдумывать защитные экраны.
#125 #352349
>>352348

>>352342

>Можно ли ее будет поддерживать


В космос никто кисель не проливал, поддерживать её не требуется. Лишь молиться чтобы в корабль не врезалась мимопылинка, ну или выдумывать защитные экраны.

Фикс, ссылка забыта.
#126 #352350
>>352332
Окей, а где еще проблематику можно поискать, чтобы с гайдом, как решать?
#127 #352354
>>352342
Держать связь сможешь, но с временными лагами. Сейчас задержка с Вояджером например около 18 часов в одну сторону, а он по сути недаавно за пояс Койпера вышел.
#128 #352356
>>352348

> какие блжад проблемы в том чтобы выпустить из специализированного фонарика тонкий луч света, поправляя его мощность и всё остальное на подлёте чтобы было похоже на свет звезды


> докапываешься

#129 #352365
Вселенная конечна и имеет расширяющиеся границы, ведь так? Или до сих пор в научной среде идет мысль об её бесконечности?
#130 #352371
>>352348

>если просто обить их неотражающей материей и выключить всё оборудование


Все равно корабль каждый будет иметь свою температуру. В инфракрасном будет светиться как елка, до фона никак не охладишь никакими йоба материалами. Это как в фольге печься - снаружу тепло выходить небодет, а вот внутри сжаришься. Потому только обманки, но смелые и наглые - типа плазменных проекторов, которые ебашат, но за их раскаленнкой не различить деталей.
#131 #352372
>>352365

> Вселенная конечна и имеет расширяющиеся границы


наблюдаемая - да, но со временем границы возможно упадут до локальной группы
на предложение "глянуть что там в ненаблюдаемой вселенной" - могу прозондировать хуём твою ротовую полость разве что
#132 #352373
>>352372
Ненаблюдаемая вселенная уже создана или создается по мере расширения?
#133 #352376
>>352373
Никто пока не знает конечная вселенная или нет. Край ,наблюдаемой нами вселенной, находится там, откуда успел до нас дойти свет. Этот край находится в 47 миллиардов световых лет от нас, свет от него шел 13 миллиардов лет. Вселенная расширяется с ускорением, поэтому свет всего ,находящегося за этим карем, до нас уже никогда не дойдет. По этой же причине край наблюдаемой вселенной становится ближе. Пройдут года, века и вечности ,и ньюфажные обитали нашего Млечного Пути/Милкомеды уже не смогут наблюдать соседние галактики и будут задаваться вопросом : "А хуле вокруг все темно?". Никакая информация из-за края вселенной до нас не дойдет. И по поводу расширения, из-за расширения вселенной новая материя не создается, а лишь увеличивается пространство между существущей, вселенная расширяется везде, а не у каких-то краев.
#134 #352378
>>352373
>>352376
Вот да, чтобы он получше понял: есть теория того, что вселенная замкнута сама на себя. Она даже довольно распространена. И расширяется как надувающийся шарик, если вдоль шарика ходить то всегда будешь возвращаться туда, откуда шёл, тут так же. А ещё я где-то слышал теорию, что если сделать кружочек, то когда вернёшься всё в тебе будет инвертировано слева направо, но это я просто слышал, фиг знает, почему и правда ли.
#135 #352394
>>352328

>остановила расширение Вселенной хотя бы локально


Согласно современным представлениям - нет. Расширение вселенной со временем распидорасит всё, даже твою мамашу, даже небо, даже Аллаха.
#136 #352395
Объясните энтропию не термодинамическими терминами. Вот чтобы сука любой даун мог понять
#137 #352398
>>352395
Чем меньше предметы кучкуются — тем меньше энтропия. Чем более равномерно они распределены по какому-либо объему — тем больше. Представь, что все атомы во вселенной на абсолютно одинаковых расстояниях абсолютно равномерно распределены по объёму вселенной. Энтропия в таком случае ММММаксимальна. В случае термодинамики повышение энтропии вселенной это когда всё горячее рассеивает свою энергию в пространство, везде энергии становится понемножку и в результате везде холодно. Соответственно, чем меньше энтропии, тем больше крупных источников энергии и тем эти источники мощнее.
#138 #352399
>>352398

>Чем меньше предметы кучкуются — тем меньше энтропия.


Не меньше, а больше. Больше кучкуются — меньше энтропия.
фикс
#139 #352400
Поясните, что за позиция в науке, что, мол, вселенная так странно устроена, что куда бы вы не отправились, двигаясь строго пол прямой, не поворачивая ни назад, ни даже в бок - в любом направлении вселенной с любой скоростью, вы всегда всё равно рано или поздно вернётесь в начальную точку, откуда вы там выбрались изначально.
Как так?
#140 #352401
>>352398
Хотя я и тут обосрался. Скорее, чем больше беспорядка будет в движении атомов во вселенной, тем больше энтропия. "все атомы во вселенной на абсолютно одинаковых расстояниях абсолютно равномерно распределены по объёму вселенной" — это, скорее, порядок и энтропия в нём минимальна. Чёрт, я и сам благодаря этому вопросу учусь понимать энтропию, извиняюсь за свои нелепые потуги.
#141 #352402
>>352401
Кароч энтропия это пиздёж и никто нихуя в ней не понимает.

>>352400
Странная у нее клава. Как такая раскладка называется?
#142 #352403
>>352400
>>352378
Замкнутость. Можешь представлять это замкнутостью через четвертое измерение. Будто мы на двухмерной поверхности шара, который замкнут сам на себя, только у поверхности шара два измерения и замкнут он через третье, а у нас три.
#143 #352404
>>352402

>Кароч энтропия это пиздёж и никто нихуя в ней не понимает.


Кароч ты нихуя не понял, поэтому "врёти".

>Как такая раскладка называется?


Такая раскладка называется "цифровой блок слева, а не справа"
#144 #352405
>>352402

> Как такая раскладка называется?


Рисунок > Отразить/повернуть > отразить слева направо
#145 #352406
>>352404

>Кароч ты нихуя не понял, поэтому "врёти".


Так никто нихуя не понимает. Выдумали какую-то математическую хуйню как обычно и дрочат на нее, а в реальности она никакого смысла не имеет.
#146 #352407
>>352406
Имеет. Мера беспорядка. Функция состояния в физике, физическое определение тут в принципе мало кто понимает, поэтому тебе объяснили "длятупых" как ты и просил, и такое объяснение, само собой, неполно. Это функция, ей пользуются, её понимают, но тут тебе полное объяснение никто давать не будет потому что ты его не поймёшь.
#147 #352408
>>352407

>то функция, ей пользуются


Да нихуя, она нахуй не нужна, никакой пользы от такого маняпонятия, как и от цифры гугол и половины математики. Понавыдумывали говна чтобы просто мозги себе поебать, только и всего.
#148 #352409
>>352408
Хорошо солнышко, как скажешь, теперь уходи.
#149 #352410
>>352408
Скажи ещё, что бесконечностью не пользуются. Такая же функция, как и энтропия. Есть ещё деревенское понимание, его мы выше объяснили, им даже мы можем пользоваться. Если я теперь скажу, что

>Понавыдумывали говна чтобы просто мозги себе поебать, только и всего.


Ты путаешь причину и следствие. Всё это говно выдумывают чтобы минимизировать количество недопониманий между учёными, чтобы у каждого процесса было такое определение, которое можно вставить в одну из формул и оно будет подходить к общей картине мира, а не чтобы подрочить на свою офигенность. Само собой, энтропия — узкое понятие и используется мало где назвиздел, открой википедию — там тебе и информационная энтропия, и, блжад, какой там только нет её, скажи ещё после этого что ими не пользуются, но этому понятию дали чёткое определение потому что этого определения не было, а не просто потому что.
>>352409
Двачую, что ты здесь вообще делаешь с таким подходом?
#150 #352412
>>352410

>Если я теперь скажу, что


Блжад, забыл закончить предложение. Ну, я могу что угодно сказать. Что у твоего соседа, который носки по дому разбрасывает, энтропия носков больше, или что энтропия вселенной растёт и рано или поздно нам звиздец из-за этого. И я буду применять это понятие потому что оно удобное а не чтобы повыделываться. Капиш?
#151 #352414
>>352395
Ну стоит у тебя кружка чая в холодильнике.
Распространение тепла от чая по всему холодильнику - это увелечение энтропии. Если бы наоборот холодильник остыл до еще более холодной температуры и отдал тепло чаю, нагрев его, это было бы увелечение энтропии, что является невозможным.
#152 #352415
>>352414
Это было бы уменьшение энтропии*
#153 #352430
>>352414
Почему нельзя нагреть чай в камере потратив на это энергию?
#154 #352439
Ананас, зачем на космических кораблях устанавливают солнечные панели? Почему даже Шаттлы умудрялись летать без (и каким образом? Жгли топливо? Химически окисляли какие-то топливные брекеты и питали напрямую генератор?), а на какие-то автоматические беспилотные транспортные Прогрессы нужно обязательно их устанавливать?

Почему совки в своё время отказались от аккумуляторной схемы и прицепили панельки? Был выигрыш по весу и в итоге по деньгам?

Неужели современная промышленность не может впихнуть половину Теслы в Прогресс, за счет этого, возможно, отказаться от ряда процедур по зарядке батарей, возможных рисков в связи с нераскрытием и прочих неудобств? Или выигрыша не будет?

Прогресс же может зарядить свои батареи от МКС?

навеяно 2-х витковой схемой полета к МКС
#155 #352443
>>352439
На шаттлах топливные ячейки стояли:
https://www.nasa.gov/topics/technology/hydrogen/fc_shuttle.html
Такие штуки наши на ЛОК хотели поставить, но почему после отмены программы это дело никуда это не пошло - хз.
За остальное не поясню, ибо не знаю же.
#156 #352445
>>352439

>возможных рисков в связи с нераскрытием


А, то есть рисков того, что аккумулятор полетит, нет?
Я лично не понимаю таких вопросов. Панель = куча халявной энергии, просто сколько хочешь её, какие аккумуляторы, какая топливная зарядка? Зачем?
#157 #352446
>>352445
Я возможно что-то путаю, но СБ же всё равно не могут напрямую питать системы корабля - они заряжают буферные батареи, а те уже раздают лепестричество. То есть аккумуляторы всё равно тащить приходится, пусть и не такие жирные. Разве не так?
#158 #352447
>>352445
Ну аккумуляторы там и так есть, не надо городить отдельный конструктивный узел, а просто расширить емкость существующего. А насчет хялявы: ко мне в детстве мужик лез с предложением москвич ржавый выпотрошенный труп за 1000 рублей продать напоить меня и изнасиловать. Нахаляву. Но мне 1000 была дороже, т.к. москвич был не нужен.

Ты что, домой собрался это электричество вести?
#159 #352448
>>352447

>за 1000 рублей продать


купить /очевидный фикс
#160 #352451
>>352446
>>352447
Ну, я всегда полагал, что аккумулятор — это жирно. СБ тоже, но если летать надо много, то аккумулятор получился бы слишком жирным. На такие взлетел-упал я тоже не понимаю зачем панели ставить. Разве буферные батареи делают тяжелыми? Вроде рассчитывают так, чтоб только не разрядились пока планета Солнышко загораживает.
#161 #352461
>>352430
Ты имеешь в виду нагреть чай в холодильнике, сделав при этом холодильник еще холоднее? Гугли Демона Максвелла. Это тоже самое, если ты запустишь газ в вакуумную камеру, и он вместо того, чтобы занять все пространство, соберется в один квадратный сантиметр
#162 #352469
>>352350
я думал ты сам умный. Поищи статьи, почитай. Курсач же пишешь, тебе все равно самому придется делать кучу. Если не знаешь где искать статьи, то вбей в поиск "как искать научные статьи" или "где"
#163 #352473
почему не бывает такого, что дальние планеты вращаются быстрее ближних?
что должно быть, что бы такое работало? антигравитация звезды? антиматерия? тёмная материя?
#164 #352477
>>352112
Дело не в давлении, а в плотности атмосферы. И до кучи в составе.
#165 #352478
>>352105
1. Может. Но нахуя?
2. Как ты себе это представляешь?
3. Можно. Но не взлетит. Точнее, взлетит, но то, что ты поместил туда в качестве полезной нагрузки, даже мать родная не узнает
>>352113
Алюминий писать научился, а баллон ещё нет
#166 #352479
>>352287

>какую роль играют фотоны в строении атома


Не дают ему разлететься на составляющие (фотон - переносчик электромагнитного взаимодействия).

>не берутся же они из неоткуда


О, сколько нам открытий чудных...
ИЗ НИОТКУДА, БЛЯТЬ!
>>352288

>что конкретно мы видим на фотографиях атомов


Покажи хоть одну
>>352289
А его итак нельзя увидеть в микроскоп
#167 #352480
>>352312

>ядра отдельно от атомов только совсем недавно научились "фотографировать"


Ммм?
#168 #352481
>>352327

>Цвет железа - понятно какой


Автохуй. При тяготении в десятки и сотни раз больше земного могут существовать совсем неебические аллотропные модификации железа. Предсказать, какого они цвета будут невозможно. Разве только посчитать квантами.
#169 #352482
>>352473
Потому что сила притяжения ослабевает с расстоянием, а центробежная сила увеличивается с ростом скорости.
Это невозможно. Не будут вокруг неё вращаться. Всё равно имеет положительную массу- разницы нет. То же самое.
#170 #352483
>>352473
если внутренняя планета вблизи горизонта событий, то для удаленного наблюдателя она будет вращаться медленнее внешней
image844 Кб, 886x591
#171 #352484
>>352371

>плазменных проекторов, которые ебашат, но за их раскаленнкой не различить деталей

#172 #352486
>>352400

>Поясните, что за позиция в науке


Эта позиция исключительно в твоей пустой голове - Шок Сенсация Скачать бесплатно без СМС
Если бы у тебя мозгов было чуть больше, чем у курицы, тот-же гугл тебе бы подсказал, что: "
По результатам наблюдений обсерватории Planck при учете данных реликтового излучения (температуры, поляризации и линзирования), определено, что параметр кривизны пространства близок к нулю в пределах малых ошибок: ΩK = -0.004±0.015, — а с учетом данных по распределению скоплений галактик и измерений скорости расширения по данным о сверхновых типа Ia параметр ΩK = 0.0008±0.0040. То есть Вселенная плоская с высокой точностью"
Зад свой впереди, если быстро быстро двигаться, ты не увидишь. И в то-же самое место не вернешься. Пространство плоское.
#173 #352498
Задавал вопрос про энтропию. Почитал ответы. Вообщем больше тепла > больше энтропия, меньше тепла < меньше энтропия. При этом энергия должна быть распределена по всей вселенной, так?
#174 #352510
>>352498
О б-же...

>больше тепла > больше энтропия, меньше тепла < меньше энтропия


Количество тепла всегда одинаково. Но если тепло кучкуется или находится в любом ином более-менее упорядоченном виде — энтропия меньше. Чем больше тепло рассеивается по пространству, чем больше в его поведении хаоса и неопределённости, тем энтропия больше. Если сейчас все звёзды взорвутся и тепло разлетится во все стороны — энтропия резко вырастет потому что движение тепловых частиц станет более хаотичным и менее предсказуемым. И это всё только тепловая энтропия. Само по себе понятие "энтропия" шире и может быть применено не только к теплу, но если о тепле, то вот я выше расписал.
#175 #352518
Что там с китайским FAST-ом? Сколько еще тестить будут?
#176 #352523
Спиннеры в космосе уже крутили?
#177 #352524
>>352523
Ещё до того, как это стало мейнстримом.
#178 #352525
Если прям сейчас на орбиту земли поместить объект с массой земли. Ну скажем венеру. А луну передвинуть чуть подальше. Дабы сохранить стабильность системы надо ли ускорять новую двойную систему относительно солнца? И будет ли система двигатся стабильно хотя бы миллиард лет?
#179 #352526
>>352525

>фик


Поместить объект надо на орбиту луны.
#180 #352527
>>352525
Не надо ускорять, орбитальная скорость на одном расстоянии одинакова что у чайника, что у планеты.
#181 #352528
>>352525
Зависит от того, как ты поместишь этот объект. Если он возникнет никоткуда и будет иметь нулевую скорость относительно солнца, то да, придется двойную системы разогнать. Если он имеет нулевую скорость относительно Земли, значит он уже движется по орбите и ничего не надо разгонять.
#182 #352532
В матаче игнорят, спрошу тут. Непрерывно ли множество неалгебраических чисел? И если да, то как это вяжется с фактом, что алгебраические повсюду между них? Какие веские основания не принимать этот факт как контраргумент к континиум-гипотезе (которая сама по себе на верну принята)?
#183 #352535
Котятки, я вот что не понимаю: Луна теряет энергию из-за гравитационного взаимодействия с Землёй - приливы, например. В результате мы имеем приливную синхронизацию Луны. Также известно, что раньше Луна была ближе и по причине этого самого гравитационного трения она удаляется. Но почему? КСП научило меня, что если корабль теряет энергию, то он должен шлёпнуться на родительское тело.
#184 #352537
>>352535

> гравитационного трения


> КСП научило меня


не тому научило
#185 #352539
>>352537
Сам нашёл ответ а ты злой, плохой,
пусть тебе будет стыдно
#186 #352540
>>352539
ну ок
#187 #352541
>>352540
Про качество ответов в шапке не сказано
#188 #352542
>>352541
только про качество отвечающих
#189 #352553
>>352510
заебись, благодарю!!
#190 #352555
>>352535
Чуть-чуть теряет кинетическую, а в целом энергия растёт. А вот Земля её теряет.
сферическая астрономия #191 #352566
Привет, анон. Можешь накидать видео/анимаций в качестве наглядного пособия по сферической астрономии? 4 системы сферических координат, относительные движения Солнца и планет, время и т.д. Учусь на геодезиста, готовлюсь к экзамену, так что сложного матана не нужно, просто доступность и наглядность, по статичным рисункам вкуривать долго, а времени мало.
#192 #352576
>>352518
Ну так что там блять с этой ебаной тарелкой?
#193 #352578
>>352576
Это ж китайцы. Потестят и скажут, что готово. А потом выдадут на-гора тучу фейковых данных.
#194 #352581
>>352576
Что ты ведешь себя, как капризная баба? Что тебя конкретно интересует?
#195 #352590
>>352045 (OP)
Че там у vasimr'а, давно про него нет новостей.
#196 #352592
>>352581
Прошла ли Фаза тестирования и начались ли научные исследования? Будут ли также оперативно публиковаться резултаты как на сайте НАСА по образцу с Хаблом. Алсо, "дружит" ли НАСА с китайской космической программой или будут игнорить китайские результаты, а инфу придется черпать на родном сайте FAST-a?
#197 #352595
>>352590

>vasimr


К середине следующего года хотят стендовый сточасовой прожиг сделать.
На МКС пока мощности нет для постоянного отжига, да и запускать пока не собираются, хотя постоянно про это говорят.
#198 #352596
>>352595
В чем суть этого вашего васимира? Чем он от обычного ионника отличается?
#199 #352598
>>352596
До 2Н может выхлопом пердеть, что раз в 20 лучше показателей похожих по габаритам ионников; еще удельный импульс выше в десяток раз. При наличии мощного источника энергии (ага щаз) можно таскать куда более тяжелые конусы чем сейчас на дальние дистанции в разумные сроки.

Планировали на лунную орбиту им восьмитонные грузы за полгодика таскать, а вот что дальше и про другие способы применения ниче особо не пиздят.

Ах да оно ещё и долговечнее, выхлоп стенки не царапает.
#200 #352600
>>352045 (OP)
Не хочу создавать отдельный тред, поэтому спрошу здесь.

Пришла в голову такая технология изготовления защитного щита для спускаемого аппарата. Не абляционная защита, а по типу защиты шаттла - термостойкая керамика. Короче, смысл такой: изготавливается металлический каркас по форме щита, который потом будет крепиться на внешнюю поверхность аппарата - т.е. он будет съёмным целиком, не по частям. Сплав выбирается такой, что при нагреве не очень сильно меняет объём, кроме того, прочностные характеристики должны оставаться примерно прежними. Этот металлический каркас изготавливается по типу металлической сетки, вроде той, на которую крепится плитка при обкладывании домов для улучшения схватывания, причём "прутья" этой сетки жидкостно охлаждаются, т.е. представляют собой полые трубки. Затем создаётся расплав тугоплавкого вещества, трудно сказать, какого именно, т.к. очевидно это уже будет не совсем керамика, и каркас опускается в этот самый расплав, при этом работает жидкостное охлаждение. Идея в том, что тугоплавкий состав будет прилипать к прутьям, остывая, образуя фактически идеальную поверхность, которая в дальнейшем потребует минимальной обработки. Технология в целом выходит не ужасно сложной - не придётся изготавливать каждую плитку и по одной клеить, нет риска, что из-за мелкой щели (которая могла бы быть при укладке плитки как дефект производства) или слетевшей плитки сгорит сам аппарат. Расплав можно получить в жидкостно охлаждаемом сосуде, т.е. плавиться материал будет где-то только в центре огромного его слитка, куда и будет происходить погружение. Металл каркаса не будет повреждаться из-за быстрого его охлаждения. В общем, по принципу выращивания искуственного рубина.

Теперь вопрос. Возможно ли подобрать тугоплавкий материал так, чтобы не он не трескался от нагрева при входе в атмосферу? Т.е. при нагреве он будет расширяться, будут создаваться внутренние напряжения из-за градиента нагрева и арматуры, это всё может привести к трещинам, которые в такой ситуации будут чрезвычайно опасны. Не это ли явилось причиной выбора на шаттле именно мелких плиток?

Кроме того, на такой материал возможно нанесение сверху не очень толстой абляционной защиты, чтобы прийти к компромиссу между приемлемой толщиной наплава, многоразовостью и уменьшением риска трещин в монолитном щите.
#200 #352600
>>352045 (OP)
Не хочу создавать отдельный тред, поэтому спрошу здесь.

Пришла в голову такая технология изготовления защитного щита для спускаемого аппарата. Не абляционная защита, а по типу защиты шаттла - термостойкая керамика. Короче, смысл такой: изготавливается металлический каркас по форме щита, который потом будет крепиться на внешнюю поверхность аппарата - т.е. он будет съёмным целиком, не по частям. Сплав выбирается такой, что при нагреве не очень сильно меняет объём, кроме того, прочностные характеристики должны оставаться примерно прежними. Этот металлический каркас изготавливается по типу металлической сетки, вроде той, на которую крепится плитка при обкладывании домов для улучшения схватывания, причём "прутья" этой сетки жидкостно охлаждаются, т.е. представляют собой полые трубки. Затем создаётся расплав тугоплавкого вещества, трудно сказать, какого именно, т.к. очевидно это уже будет не совсем керамика, и каркас опускается в этот самый расплав, при этом работает жидкостное охлаждение. Идея в том, что тугоплавкий состав будет прилипать к прутьям, остывая, образуя фактически идеальную поверхность, которая в дальнейшем потребует минимальной обработки. Технология в целом выходит не ужасно сложной - не придётся изготавливать каждую плитку и по одной клеить, нет риска, что из-за мелкой щели (которая могла бы быть при укладке плитки как дефект производства) или слетевшей плитки сгорит сам аппарат. Расплав можно получить в жидкостно охлаждаемом сосуде, т.е. плавиться материал будет где-то только в центре огромного его слитка, куда и будет происходить погружение. Металл каркаса не будет повреждаться из-за быстрого его охлаждения. В общем, по принципу выращивания искуственного рубина.

Теперь вопрос. Возможно ли подобрать тугоплавкий материал так, чтобы не он не трескался от нагрева при входе в атмосферу? Т.е. при нагреве он будет расширяться, будут создаваться внутренние напряжения из-за градиента нагрева и арматуры, это всё может привести к трещинам, которые в такой ситуации будут чрезвычайно опасны. Не это ли явилось причиной выбора на шаттле именно мелких плиток?

Кроме того, на такой материал возможно нанесение сверху не очень толстой абляционной защиты, чтобы прийти к компромиссу между приемлемой толщиной наплава, многоразовостью и уменьшением риска трещин в монолитном щите.
#201 #352601
>>352600
Слишком сложно и дорого. Какие задачи у подобной хуйни? В чем конкретные преимущества?
#202 #352606
>>352601
Чем сложно и почему дорого?
В чём сложность? Металлический каркас не вызовет проблем в изготовлении, технология достаточно отработанная и старая, получить расплав - тоже нетрудно, тоже отработанная технология, с помощью которой изготавливаются промышленные кристаллы рубина, например, но вообще не только. Я просто не могу слёту сказать, что ещё, но сложности я вижу только в подборе материалов. Да, понимаю, что не один такой дохера умный и наверняка об этом думали, но вот проблема самого принципа не ясна.
Задачи - это же многоразовая защита для спускаемых аппаратов. Можно составить не такую "в лоб" схему, как выше, а несколько сложнее, с дополнительными слоями, я не знаю. Преимущества: во-первых, их можно клепать десятками и это будет недорого, во-вторых, современные технологии (лазер в вакууме, например) позволяют усложнить требуемую форму, в-третьих, это бесшовная защита, т.е. риск повреждения аппарата сильно уменьшается.
#203 #352607
>>352606

> не вызовет проблем


> нетрудно


> характеристики должны оставаться примерно прежними


> Затем создаётся расплав тугоплавкого вещества, трудно сказать, какого именно, т.к. очевидно это уже будет не совсем керамика


манямир
никогда не думал что скажу здесь это слово
#205 #352609
>>352045 (OP)
Кстати, почему не делают ионников на водороде? Удельный импульс же. А то всё инертные газы и ртуть
#206 #352610
>>352609

> Удельный импульс же


именно из-за него
#207 #352611
Анон, смотри, читаю рувики про Челенджер и там написано такое.

>сообщил о том, что если погода будет слишком холодной, то уплотнительные кольца (англ. O-rings), соединяющие секции твёрдотопливных ускорителей челнока, могут выйти из строя, и возможная катастрофа может привести к гибели челнока. Божоли основывался на наблюдениях за шаттлом «Дискавери», вернувшимся из экспедиции STS-51C: по данным Божоли, первое резиновое кольцо (верхнее) на одном из ракетных ускорителей было полностью разрушено, а второе (нижнее) было серьёзно повреждено, но не было разрушено до конца; его толщина составляла 1 мм в самой слабой части


Трудности перевода? Ведь ёбнул твердотопливный изначально, который находится по бокам? как он мог проверить твердотопливный на шаттле самом? Они же отстреливаются и падают,а не улетают в космос.
#208 #352612
>>352611
ТТ ускорители приземлялись на парашютах в океан и потом транспортировались для повторного использования
#209 #352613
>>352612
Да вот и я про то же. Зачем инженеру этому тогда нужно было ждать возвращения шаттла? Значит хуево написано.
#210 #352614
>>352606
Одну плиту тяжеловато будет, не? А если тонкую, то менее надежно чем чешуей много тонких в нужных местах наклеить. Да и самолетики больше не хочет никто клепать, а абляционка и щас норм.
#211 #352625
>>352610
Непонимат. Чем меньше масса разгоняемого иона же, тем больше скорость "истечения". Пусть импульс mv на моль вещества и будет низким, но отнесённый к массе будет mv/m=v, то есть больше?
#212 #352635
>>352625
Момент разный и принципы работы разные.
Если в случае с химией ты разогреваешь компоненты до определенного температурного порога (скажем, все движки 2000 градусов, значит легче будет быстрее разогнано), то ионники разгоняют электричеством.
Скорость истечения зависит в т.ч. от напряженности, заряда и прочего. У водорода единственный электрон отнимать накладнее, чем крайний у аргона и тем более ксенона. Разница в заряде ионов водорода и ксенона тоже имеет значение.
А импульс так вообще роляет - чуть ли не вдвое больше у ксенона, чем у аргона при одном заряде.

Кароч чет такое, надеюсь кто-нибудь разжует, а то я второпях попивая кофе это насрал.
#213 #352636
А с ведром что там? Столько пиздели что на сае, что на этой доске на пустом месте (особенно вокруг того фейка, где китай собирался испытать на орбите его) и вдруг все внезапно заглохло, последние упоминания на конец прошлого года приходятся.
#214 #352638
>>352636
Потому что вселенная опять загубила норм технологию своими говнистыми анально ограниченными законами физики
ну не сработала нихрена, как обычно
Поэтому опять сидим на Земляшке и сосём бибу Ж(
#215 #352640
При пуске РН с ПН - страхуются оба + прочие риски? Или это на усмотрение клиента, может застраховать, например, только ПН?

Запуски, которые осуществляют военные - страхуются? Я помню анекдоты про страховые выплаты по ГЛОНАССам, но если это, например, военный аппарат заточенные только под военные нужды, условный спутник системы предупреждения о ракетном нападении или спутник-шпион, его тоже страхуют? По обе стороны океана?
#216 #352644
Космач, есть одно задание в универчике, на проектирование КА, ограничений особых нет, можно творить, как угодно, в пределах разумного.
В мыслях у меня уже около года КЛА для изучения Сатурна. Именно КЛА, подразумеваю, что мой аппарат будет летать в атмосфере Сатурна и проводить там всяческие научные штуки. Не аэростат, как на Веге, а сорт оф самолёт.
Так вот. Поясните за то, что там можно изучать с научной точки зрения?по мне так просто летающая йоба в атмосфере Сатурна - это уже охуенно и достаточно Очевидно, уточнение состава атмосферы, поведения атмосферы, ее явления. Читал где-то, что на Сатурне и Юпитере возможна жизнь, благодаря содержащимся там парам воды, в одном из слоев атмосферы, но нигде особо инфы не нашелхуево искал, да Можете что подкинуть по этому вопросу?
Ну и вообще, как вам идея? Ебучие транспортники, тягачи, роверы, зонды и прочая хуета - как-то скучно и приземленно, на мой взгляд, хочется йобы.
#217 #352645
>>352635
Ладно, пойду курить матчасть. Я ещё вернусь и подоёбываюсь до вас
#218 #352646
>>352640
Клиент не может страховать ракету, она ему не принадлежит. Страхуется полезная нагрузка.
Запускатор по идее вроде может свою ракету страховать, но точно не в курсе.
Воене не страхуют запуски финансово, страхование у них происходит скорее за счет того, что спутники одного типа они по несколько штук пускают а то и группировками.
#219 #352647
>>352644
Норм затея, пишешь все примерно верно.
Почитай текущую инфу про атмосферу и прикинь, выдержит ли твой воздушный шар такие порывы ветра с помощью магии сопромата нет
А в тягачах и роверах нет ничего плохого.
Можешь дюнолёт попробовать рассчитать, как НАСА. Ой, марсолёт. Или венеролёт. Или доставку образца грунта. Или доставку образца астероида.
То, что подобное когда-то делалось не делает эти йобы менее охуенными.
Вот если бы половину камней в СС бороздили роверы как нашу планету бороздят машины - тогда да.
#220 #352648
Вопрос про военные спутники: для чего они щас используются? Понятно, без конкретики, но вообще любопытно - их же постоянно пускают.
Раньше фоткать землю пускали, щас тоже, но нужно мало, ведь не надо пленку возвращать на проявку, фоткают и шлют по вайфаю.
Глонасс/гпс уже развернуты.
Спутники связи тоже уже все развернули.
Для чего нужны новые? Есть ли еще области не упомянутые выше?
#221 #352651
>>352648

>Для чего нужны новые?


Очевидно, на замену. Как морально устаревших, так и чисто технически старых спутников, исчерпавшие свой ресурс.

А так с дивана: спутники ДЗЗ (радиолокационной, радиотехнической и визуальной разведки), невообразимое количество всевозможной связи (вояки, дипломаты, земля-земля/космос-космос ретрансляторы, для вмф, для ракетчиков, для общевойсковых нужд, йобахитрые командные для дня Пи и пр.), спутники СПРН, локальные навигационные и спутники системы глобального позиционирования, спутники-платформы для обкатки технологий.
#222 #352652
>>352647

>воздушный шар


Это будет ЛА, скорее всего даже с двигателем, но это ещё обдумаю, аэродинамику посчитаю, сопромат прикину. Не вопрос. Скорости входа, массы, компоновку, модели - тоже все прикину. Там должна быть исследовательская часть и, скорее всего, она будет в виде выбора аэродинамики аппарата.
Про жизнь я все верно сказал? В Википедии нет ничего, хотя вроде раньше было.
#223 #352653
>>352651
Спасибо, разумно.

>невообразимое количество всевозможной связи (вояки, дипломаты, земля-земля/космос-космос ретрансляторы, для вмф, для ракетчиков, для общевойсковых нужд,


А почему не один стандарт?

>СПРН


Вот никогда не понимал вообще даже в теории как СПРН работает. Как можно спалить мелкую ракету на фоне колоссальных территорий, особенно если она в другом полушарии стартует?

>локальные навигационные и спутники системы глобального позиционирования


Зачем нужны локальные, если есть глобальные? И локальные же бесполезны, их бригаду надо выводить чтобы хоть как-то область покрывать, проще сразу глобально.
#224 #352654
>>352652
Про жизнь на других планетах у нас только манятеории. Что-то авторитетно заявлять можно только заспамив и хорошенько перерыв много веществ.
По мне так нихуя на гигантах нет. Может на какой-нибудь европке, венере и марсе и была протожизнь, но мы и ее скорей всего не найдем, т.к. дальше одноклеточных она не должна была успеть развиться.

А по поводу ЛА - вспомни какой ёбой был шаттл.
Да, у тебя будет поменьше, но тоже теплозащита, двигательная установка, вся лютая компоновка, выходит шибко йобисто как по мне.
Проще представить раскладной дрон который прилетает на марс в коробчонке 0.5х0.5 и раскладывает свои 20-метровые крылья из полиэтилена и дует трехметровым пропеллером истошно пытаясь взлететь, а марсианский орбитер над ним потешается.
#225 #352655
>>352653

> А почему не один стандарт?


шоб сикретно

> Как можно спалить мелкую ракету на фоне колоссальных территорий


легко, она светится ярка)

> особенно если она в другом полушарии стартует


полушарий мало, спутнеков многа

> И локальные же бесполезны, их бригаду надо выводить чтобы хоть как-то область покрывать, проще сразу глобально.


там специфические орбиты

> Зачем нужны локальные, если есть глобальные?


задоч оптимизирования не стоит
#226 #352656
Ребят, что я только что придумало.
Взять мега телескопы и смотреть на соседний дом с тёлкой Андромеду. И спектрометрами высокочувствительными мерять. Так вот, настроить их на спектр излучения при атомных взрывах и задектектить так сказать высокоразвитую циливизацию. А? прокатит?
#227 #352657
>>352656
Ты найдёшь порядка триллиона атомных взрывов.
Смысл?
#228 #352658
>>352638

Не, посоны норм сидят думоют, один ты только сосёшь, чмырик, уж эволюция позаботилась о твоих анальных разорвограничениях.
#229 #352660
>>352654
Ну, размеры шаттла туда не долетят. Посчитаю массы для начала и скорости, будут цифры - будут варианты.
Так-то предполагаю складную конструкциювозможно на полимерах с памятью формы, тормозить надувным, тепло прикинуть надо, оно не столько волнует, сколько q, хотя она зависит от плотности и будет меньше, чем на Земле. Да и тормозить надо не до 0, а до скорости когда станет возможным управляемое планирование, локальная Ду если и будет, то для коррекции/перестраховки/придумаю. Просто глупо не использовать летающий вокруг с бешеной скоростью водород и гелий.
На Марс у меня не стоит так сильно, как на Юпитер или Сатурн. Европу ещё можно, но там типовой СА с буром и оборудованием.
#230 #352661
>>352660
А, ну и конечно, орбитальный модуль ещё нужен, дабы ретранслировать через него сигнал на Землю.
Сейчас очень модно делать всё без помощи зарубежных машин, так что это будет плюс.
#231 #352662
>>352660
Тогда попрошу поделиться на доске исчислениями, даже если они порушат твою задумку с летательным аппаратом.
Если ты действительно хорошо к этому приложишь руку - создай тред, распиши матан, можешь всю свою работу даже выложить (в формате треда).
Было бы очень.
Удачи.

Не забывай спрашивать ответы и репортить прогресс, кстати.
#232 #352663
>>352662
Если я все таки утвержусь на этой теме, не завернет препод и будет результат, то создам, да.
#233 #352664
>>352663
У тебя все равно будут какие-то предварительные результаты исследования по выживаемости в ветрах сатурна, например.
А даже если завернет, то вообще просто тред про проектирование, анализ, циферки в космическоаппаратостроении немного подоживил бы подзамерзшую доску, так что в любом случае пиши, даже если не про сатурнолет, даже если про "банальный" кьюриосити.
#234 #352665
>>352638
Да мне скорее социальная сторона умервщления ведра интересна, с физикой то все понятно.
#235 #352667
>>352653

>Спасибо, разумно.



>СПРН


>Вот никогда не понимал вообще даже в теории как СПРН работает. Как можно спалить мелкую ракету на фоне колоссальных территорий, особенно если она в другом полушарии стартует?



Их должно быть несколько. Это либо геостационарные спутники, они палят постоянно, либо спутники с высокоэллиптической орбитой. Спалить не сложно, у ракеты внезапно ебический факел из жопы. Разглядеть его технически не сложно.
image337 Кб, 500x332
#236 #352668
>>352667

>ебический факел из жопы. Разглядеть его технически не сложно.


Да вы шутите. Я смотрю на обзор всех США на своем фуллхд монике и не вижу даже километровые детали. Как можно разглядеть пятиметровый факел, особенно днем? Какая матрица должна стоять на этих аппаратах? Молнии еще ебашат, тоже яркие. Города яркие. Вообще не понимаю.

А еще меньше понимаю, как сраная хуита из ржавых железок стоящая в сибири как-то может детектить запуски из америки? Пикрелейтед
#237 #352670
>>352668
Двачую вопрос.
#239 #352673
>>352672
Так все это читали. Все равно нихуя не обьясняется же.
Да, волны отскакивают и оказываются люто рассеянными в США. Хули с них взять-то там, блядь? Это я как фонариком на луну посвечу, нихера не заметно на таких расстояниях, а тем более ракету находить. Пиздёж какой-то.
800px-OTH-B.big48 Кб, 800x400
#240 #352675
>>352668

>ебический факел из жопы. Разглядеть его технически не сложно.


>Да вы шутите. Я смотрю на обзор всех США на своем фуллхд монике и не вижу даже километровые детали. Как можно разглядеть пятиметровый факел, особенно днем?


Ты ничего про другие диапазоны электромагнитного излучения не слышал? ИК, например? И не пятиметровый, а гораздо больше. Шлейф за ракетой неебический, лютой температуры. На фоне окружающей среды как в тёмном ангаре въебать пятикиловаттный прожектор.

>А еще меньше понимаю, как сраная хуита из ржавых железок стоящая в сибири как-то может детектить запуски из америки? Пикрелейтед


https://ru.wikipedia.org/wiki/Загоризонтный_радиолокатор
#241 #352676
>>352673
Ты ебанько и читал по диагонали?
sage #242 #352690
>>352657
нее, не термоядерных а именно при взрыве урано-плутониевых зарядов
#243 #352691
>>352221
Можно моар?
#244 #352692
>>352668
Я только один способ знаю: на просвет через атмосферу, т.е. спутник должен смотреть почти за горизонт. Факел на фоне темного космоса очень легко различим, говорят, так не только МБР можно засекать, а чуть ли не самолеты.
#245 #352705
>>352692
Это один из способов, но не единственный.
#246 #352706
>>352690
Очень маленький пук у ядерных хлопушек. Время взрыва, именно вспышка и плазменный шар, рассеиваются довольно быстро. Слишком ебовая чувствительность нужна.
е240 Кб, 1200x881
#247 #352707
>>352107

>фотон безмассовая частица, дятел


Следовательно у фотона нет энергии? Ведь сколько не умножай на ноль, нихуя не получится. Кто-то из вас пиздит, либо Эйнштейн, либо Планк, либо ты.

Эйнштейна с Планком я знаю, а вот ты что за хуй? Залогинься, мразь.
15020980989970797 Кб, 800x800
#248 #352709
Поясните за облако Оорта.
Я так понял, облако Оорта - самая манятеоритеческая фантазия в нашей солнечной системе, и что постулат о существовании этого облака строится чисто на том, что есть некоторые долгопериодические кометы СЛЕДОВАТЕЛЬНО где-то за Плутоном есть огромная куча мелких и крупных метеоритов, астероидов из камня, водяного льда, газа и пыли, прям зуб даю, инфа соточка, кто если не облако Оорта?
А какие есть ещё доказательства существования этого облака кроме манятеории без пруфов? У других звёзд что-то при всех визуальных, спектральных и радионаблюдениях никаких подобных облаков ни разу не обнаружили.
#249 #352710
>>352709
Ты очень хуево понимаешь как работает наука. Манятеории у тебя в башке пустой.
Теория об облаке Оорта построена не только на нескольких долгопереодических кометах.
Отсутствие наблюдательных данных легко объясняется размерами массой и температурой комет, а так же невероятным, для осознания скудным воображением, объёмом пространства в котором эти кометы находятся. Если облако Оорта мы не можем пока детектить у себя во дворе, то отсутствие наблюдательных данных у других систем не аргумент.
Если совсем по существу, то да, Облако Оорта пока не найдено. Однако эта теория отличается от суслика, которого не видно, но он есть.
фотошоп-MoodyBlue-песочница-35814381,5 Мб, 1274x614
#250 #352711
>>352710

> Теория об облаке Оорта построена не только на нескольких долгопереодических кометах


Хорошо, какие ещё есть обоснования в пользу облака Оорта?
Я на полном серьёзе этим интересуюсь и хотелось услышать больше инфы и вводных данных для анализа.
image3 Кб, 196x39
#251 #352715
>>352707

>Кто-то из вас пиздит, либо Эйнштейн, либо Планк, либо ты.


>запостил упрощенную формулу картинкой которая нерелевантна для безмассовых частиц

#252 #352716
>>352715
р в твоей формуле импульс, правда? А формула импульса, внезапно, включает в себя МАССУ.

Школьник, отсутствие массы ПОКОЯ не означает отсутствия массы вообще. И формула не упрощённая, а фундаментальная, благодаря которой ты можешь рассчитать массу летящего фотона.
#253 #352717
>>352711
Только прикидки и ожидания, ничего конкретного мы не можем в ближайшее время узнать.
По прикидкам масса облка составляет пять масс земли. Поскролль хороший симулятор космоса вроде хотя бы спейсэнжина чтобы осознать размеры.
Это небольшой остаток после изначального распидораса облака из которого образовалось Солнце, та часть, что осталась в гравитационном влиянии его и скомпоновалась в кометы и пыль.

Мысли о том, что это какая-то колоссальная ебола окружающая СС - заблуждение. На деле это мало отличается от межзвездного пространства (более того, оно уже им считай и является).
Облако Оорта это область, в которой находятся афелии долгопериодических комет. Которых люто мало.
#254 #352718
>>352716

>Школьник


Не надо тут гиперкомпенсации.
И приёбываться к словам не надо. Под безмассовыми и подразумеваются частицы с нулевой массой покоя.

>И формула не упрощённая, а фундаментальная, благодаря которой ты можешь рассчитать массу летящего фотона.


Рассчитай, будь добр. Используя только E=mc2.
И я признаю, что обосрался и вообще школьник.
#255 #352719
>>352718

>Используя только


Ке-ке-ке, а чего это только? Используя эту формулу и формулу Планка мы легко и беззаботно узнаём массу летящего фотона.
#256 #352720
>>352719
Ну дык-с.
Потому я и привёл эту формулу >>352715 которая как раз применима. Простая Емс2 сама по себе - нет.
#257 #352722
>>352717

> По прикидкам масса облка составляет пять масс земли. Поскролль хороший симулятор космоса вроде хотя бы спейсэнжина чтобы осознать размеры.


Ага, а кто и как делал эти прикидки? Помнится мне, лет 20 назад все учёные обоснованно прикидывали, что к 2010 году все нефть кончится и начнётся Третья Мировая за бензин.
Я вот тоже прикидывать могу и заявляю, что масса облака соответствует не ~5 массам Земли, а 800 триллиардам. Просто разбросаны они по всей галактике и вообще вся галактика - облако Оорта Солнца. Я сказал! Вот такие у меня прикидки. Считай это за истину, отстаивай эту позицию в спорах. Я же прикинул.
#258 #352723
>>352722
Ты не так понял.
Ты услышал: "эй, парни, там кароч облако оорта обнаружили, оно хуй знает че такое, но мы вам не пиздим верти нам ))))"
А на деле: в пределах СС есть планеты. астероиды, кометы. А у некоторых комет лютые афелии. Для зоны из которых эти дальние кометы берутся и дали название "облако оорта". От межзвездного пространства оно читай ничем не отличается. Это лишь номенклатура.
Это как пытаться оспаривать "Это не парк имени горького, это вы придумали".
Дали название зоне из которой берутся дальние кометы.
Это не какое-то настоящее лютое облако которое с нихуя выдумали. Сперва были замечены кометы с большим периодом, потом дали название.
Так понятнее, надеюсь?
#259 #352751
>>352716
Так называемая "релятивистская масса" - устаревший термин, применявшийся в основном в СССР и предназначенный для удобства записи формул (формулы для импульса и т. д. оставались теми же, что и в ньютоновской физике), однако не несущий в себе физического смысла. Во всём цивилизованном мире от релятивистской массы отказались, так как она вводит двоякое трактование термина "масса" и вносит путаницу, но пост-совковые маньки продолжают ею пользоваться в силу инертности системы образования и ограниченности мышления.
#260 #352757
>>352751
Можно было не так грубо его опрокидывать.
Как нынче принято говорить? Инвариантная масса?
#261 #352779
>>352723
Да понятнее, но видимо ты не понял моей позиции. Есть, значит, эти самые долгопериодичные. Они откуда-то берутся. Обычно и как правило облако отображается именно как полая сфера:
1) Поверхность, цельнаяравномерно распределённая и равномерно содержащая по внешней стороне праздно слоняющиеся одиночные объекты.
2) Внутренняя часть облака, состоящая из солнечной системы от солнца до где-то Нептуна-Плутона
3) Внутренняя часть сферы, включающая две полые сферы - ну как полые, не замеченные в наличии тех самых оортоидов - и пояс, разделяющий "полость" на две части по экватору солнца.

Итак, по 2 пункту у меня претензий нет - тут всё неоспоримо.
По 1 пункту следующая претензия - кто и почему, прихотью чьей левой пятки, на основании чего указывает границы (пускай и условные) расположения пределов внешней стороны сферы облака Оорта? Кто сказал, что оно дальше обрывается? почему оно именно здеся, а тама? Почему тогда весь тот космохлам не барахтается именно на гелиопаузе и быть радиусом соответственно гелиопаузе? На каких основаниях? Почему бы не считать, что наше облако Оорта распрастраняется ещё дальше?
И самое батхёртное для меня пункт 3 - кто сказал, что это облако, чьё наблюдение существует исключительно на паре долгопериодичных, как я гляжу, имеет именно такую форму? Я вот, например, утверждаю, что эти камни не прилетают к нам откуда-то из-за Нептуна, а наоборот, от нас куда-то туда ныряют. Где мои деньги, Лебовски? обоснования и пруфы? Всё, что я вижу, так это высоснную из пальца огромную комплексную теорию, основанную даже не на домыслах, а всего лишь на одном.
Ты сказал, измерили массу облака Оорта. Как? Вот я тебя спрашиваю - как её измерили? Видеть не видели, т.е. визуально наличия не подтврдили, границы не обределили, туеву хучу объектов, создающих хоть бы облачко. которое можно кластерно соотнести к общему Оорту - нету нихуя.
#262 #352782
Советы с конца 60-х по 80-ые насбивали чуть ли не десяток космических аппаратов, причем если верить мемуарам, продвинулись достаточно далеко, чтобы не оставить спутнику-мишени времени на маневр, что, надо полагать, и послужило поводом к подписанию договора о запрете подобного оружия в космосе.

Есть ли подобное оружие у современной России или всё безбожно проебано? Если завтра война, Кулигины будут прикручивать к существующим ракетам необходимое оборудование или есть какое-то штатное решение?

Современная связь со спутниками осуществляется в режиме реального времени? Химические реактивные двигатели на военных аппаратах могут успеть внести какую-то существенную корректировку траектории полета, если, предположим, у них есть 2 часа на работу? Сколько может составить погрешность между расчетным и реальным положением цели? Километр? 10? 100?
1025047359 Кб, 1000x560
#263 #352796
Там это, водолаза декомпрессией надуло. Космонавтам на заметку.
#264 #352797
>>352796
Хули он живой?
#265 #352799
>>352797
Хуй знает, врачи говорят весь азот вокруг мышц собрался и он выжил, хотя сейчас испытывает неибические боли.
#266 #352813
может ли так получиться что мощная вспышка на солнце приведет к выводу из строя абсолютного большинства спутников на орбите, но при этом на земле масштабного ущерба современной промышленности не вызовет? тоесть грубо говоря человечество останется с нынешним уровнем промышленности и технологий но без спутников на орбите? по крайней мере без геостационарных? я плохо себе представляю магнитное поле земли и энергию от вспышек на солнце по этому не знаю, защищена ли в равной или в различной степени электроника на геостационарной орбите, низкой орбите и земле
#267 #352815
>>352813
Если будет такой выброс корональной массы, что придёт пизда большинству спутников, то, поверь, тут на Земле будет такой анал-карнавал, что потеря спутников покажется не особенно большой проблемой. Спутники хоть защиту имеют, да и заранее их можно отключить/отправить в гибернацию для снижения вероятности ущерба, а вот на Земле нихуя такого нет. Многие системы работают 24/7/365 и ни они, ни дублирующие их защиты не имеют в 99% случаях.
#268 #352823
>>352813

А буря вообще вредна? Ну там ПеКа сгореть может? Голова болеть? Сеть замкнет? Или нет? Даже если Солнце еще ебанет пару раз и обновит максимум наблюдений.
#269 #352825
>>352796
Не понял, это его собственное давление чуть не порвало? С ним всё в порядке?
#270 #352827
>>352825

> это его собственное давление чуть не порвало?


Эм, ну, короткий ответ - да, длинный ответ - нет. Потому что так уж устроено в нашем мире, что азот - лёгкий газ и тогда, когда остальным уже норм, ему ему уже неймётся и хочется рваться наружу. Из-за этого явления и погибли первые акванавты. Чтобы азотиум оставался в теле всплывающего, не разрывая клетки и не кипятя кровь, всплывающий плавец обязан проходить процедуру декомпрессии. Чем большая была глубина спуска и чем больше было давление, тем дольше будет декомпресия. На простом примере будет что-то типа такого: утром выходного дня в большом дом с многодетной семьёй взрослые ещё спят, а дети уже проснулись и хотят играть-буянить (это кипение азота). Если дети дорвутся, может случиться всё что угодно, хоть дом спалить. Во избежание неприятностей нужно компенсировать убегающее давление азота. А декомпрессия в таком случае будет будильником, вынужденно пробудящим спящие прочие газы перестать валяться и валить заниматься делами по дому, попутно смотря за мелкими. Ещё более простой пример: пастух и овцы.

> С ним всё в порядке?


Нет. Он раздулся. Микроразывы тканей как минимум. В худшем случае некроз.
(Автор этого поста был предупрежден.)
#271 #352828
>>352823
На самочувствие влияет. У всех по разному.
Пека не сгорит, конечно от нынешней, но если ебанет как при событии Кэррингтона и калькуляторы наебнуться могут. Но прогнозировать что-либо тяжело.
#272 #352829
>>352827

> В худшем случае некроз.


он бы тогда не стоял скорее всего
#273 #352830
>>352828

> Но прогнозировать что-либо тяжело.


а там эксперинт какой-нить эквивалентный на малом масштабе никто не проводил
#274 #352831
>>352830
типо вопрос
#275 #352832
>>352829
Некроз не пуля - в момент не убивает.
#276 #352836
>>352832

> не убивает.


но побочки много
#277 #352839
>>352830

Проводили в марте 1989 года ебануло около 60% Кэррингтона.

Норм. В Канаде и на севере США покоротило немного. Наебнулся пяток самых отсталых спутников. Полярное сияние в Крым няш-мяше и Мексике.

Пока вроде даже до сияния в Крымняшмяше не доходит текущий.
#278 #352840
https://ru.wikipedia.org/wiki/(99942)_Апофис

>13 апреля 2029 года, в пятницу, астероид 99942 Апофис пройдёт на расстоянии около 37 тысяч километров от Земли.


>Диаметр 325±15 м


>Масса 27.000.000.000 килограмм


Эту хуёвину разорвёт на куски гравитацией земли?
#279 #352841
>>352839
где про это почитать?
#280 #352848
>>352840
Как подсчитать, разорвет ли камень или нет приливными силами?
#281 #352849
>>352840
Нет, далековато. Не разорвет.
#282 #352852
>>352827

> (Автор этого поста был предупрежден.)


А вот это сейчас обидно было
#283 #352853
>>352852
тут принято дрочить на ракеты, скорее всего из-за этого
#284 #352855
Чем грозит вчерашняя/сегодняшняя солнечная вспышка ?
#285 #352856
>>352855
Головными болями и херовым самочувствием у метеочувствительных людей. Местами перебои с радиосвязью.
#287 #352859
>>352828

а можно както защитить пека? в клетку фарадея там засунуть или чонить такое? или заземлить както? или полиэтиленом обмотать в 10 слоев?
#288 #352860
>>352853
Ну так все изредка ВЫСТАВЛЯЮТ СВОИ РАКЕТЫ ПО ГОТОВНОСТИ, а у некоторых даже бывает незапланированный ПУСК. Что же это, повод ТОПЛИВО запрещать?
#289 #352861
алсо, сколько времени занимает это прохождение корональной массы? я в физике слаб, но насколько я понимаю, эта история опасна только для тех спутников и электроники на земле, которые в момент прилета будут повернуты к солнцу? а та сторона земли которая отвернута от солнца будет в этот момент, там будет всё норм и пека не сгорит?и спутники которые в это время будут за землей тоже не сгорят? или нет?
#290 #352864
>>352859
Твой пека дома, за двойным стеклом и за бетонной\кирпичной стеной вряд ли пострадает.
Бесперебойник купи, что бы не наебнулся ПЕКА, если вырубится свет.
#291 #352866
>>352861

>алсо, сколько времени занимает это прохождение корональной массы?


Если до этого не было выбросов и это первый, то дня два-три, четыре максимум. Если были выбросы, пусть и небольшие, то за сутки может долететь.

>эта история опасна только для тех спутников и электроники на земле, которые в момент прилета будут повернуты к солнцу? а та сторона земли которая отвернута от солнца будет в этот момент, там будет всё норм и пека не сгорит?


Нет, хуево будет везде, говна прилетит много и шитшторм будет хуярить пару суток, пока не пойдет на спад.
#292 #352869
>>352848
Очень много неизвестных. Нужен состав, плотность, температура, внутренняя структура, скорость вращения, расстояние, траектория, скорость и ещё черта лысого. Очень грубо можно посчитать по пределу Роша для твёрдых тел, но это будет очень хуевый подсчет.
Апофис по средней плотности не хуже гранита. Вряд ли его земная гравитация вообще способна разрушить.
#293 #352873
>>352869
А примерно с каких размеров земля/юпитер/солнце распидорасят каменюку приливными силами? И как далеко?
#294 #352879
>>352873
Земля размером с Марс и атмосферой как у Марса способна это сделать. в зависимости от... ну, всего. Траектории, скорости, рыхлости/вязкости, химсостава, подверженности влиянию ЭМП планеты, пролёту около планеты по солнечной или теневой стороны, высоте пролёте, наличии планетарных спутников и их положении и т.д., и т.п., и прочее.
#295 #352880
Cлабеет ли магнитное поле земли от солнечных бабахов? А то вот нас вспышка жжёт, жжёт и всё поле прожжёт - а если через ещё одна бомбанёт, пиздец нам?
#296 #352886
>>352880
Магнитному полю-то что? Оно сидит себе и соберает все заряженные частицы которые видит. Но не все может собрать.
#297 #352888
>>352880
Двачую этого >>352880. Земля скорее как паук, сидит в центре своих магнитных и гравитационных волн и ловит все мимочастицы, которые может и их всасывает за обе щёки
#298 #352902
>>352888

>за обе щёки


Минишиза:
Писдос, это получается Солнце на Земляшку малафит корональными выбросами, а Земля сглатывает и просит ещё? Офигеть мы на зашкваренной планете живём..
#299 #352903
>>352691
это персонаж этой девушки
https://twitter.com/ashcozy/media
#300 #352909
Аноны, допустим у нас есть идеальная ракета на теле без атмосферы.
Мы можем рассчитать её дельту по формуле циалковского вся хуйня.
Но естественно, в орбите её скорость будет меньше.
А теперь сам вопрос: как без симуляции рассчитать гравитационные потери? Насколько я понял, это можно без заёбов сделать аналитически, а всякие йоба-модели юзают из-за эффектов, которые я не буду учитывать: драг, поворот вектора тяги для маневрирования и прочие неравномерности гравполя.
#301 #352911
>>352880

Нет. Оно не может ослабеть. Ослабеть может только источник магнитного поля. Но ему похуй.

Кстати, нас защищает милипиздрическое поле. Оно слабее, чем поле в катушке компрессора в твоем холодильнике! По идее, еще лет 100 и мы сможем пилить искусственное поле. Например на каком астероиде. Для добычи ништяков.
#302 #352914
>>352902

>Офигеть мы на зашкваренной планете живём.


Чтобы ты родился из пизды — в неё нужно засунуть хуй. Все мы шкваримся, когда рождаемся.
#303 #352917
>>352903
Спасибо.
#304 #352922
А если б в Земле не было радиоактивных элементов, она бы уже остыла внутри?
#305 #352933
>>352922
нет есть ещё нагрев луной
#306 #352939
>>352922
>>352933
А ещё она остывает очень медленно, медленнее чем считалось. Конвекционные потоки двигаются с геологической скоростью.
#307 #352940
>>352902

Говорит самое зашкваренное хромосомное чмо на всём сасовостоке, орамболлирация.
#308 #352944
>>352306
Чёт ты пиздишь. Не разбираюсь, но анус ставлю на то, что бактерии зародились пару миллионов лет назад, а сутки в 4 часа были пару миллиардов лет назад.
#309 #352945
>>352944
А нет, он прав. Ну и ладно.
Хули так долго люди появлялись?
#310 #352946
>>352922
Гугли "гравитационная дифференциация недр".
#311 #352950
>>352945
Поздно, Мань. Твоё очко уже в зрительном зале.
#312 #352969
>>352909

>как без симуляции рассчитать гравитационные потери?


Посчитай идеальный взлет, посмотри/посчитай реальный взлет, вычти.
#313 #352970
>>352911
Оно на всю долбаную планету в отличие от холодильника.
#314 #352988
>>352969
Я как бы хочу аналитически найти решение и потом только сравнивать. Формулу которая с функцией угла/времени нашел, поколупал, но расхождение в 200м/с (10%, дохуя) вышло, и явных дырок не обнаружил, на лм-мы НАСА массы, бернтаймы и прочее ведь выложила в избытке.
Может Антон тут подкинет более лучшее что-то.
#315 #352989
>>352988
Блядь, сраный телефон. Нувыпонели.
#316 #353012
Есть ли примерные расчеты, а лучше симуляции, куда улетит Луна с орбиты Земли?
16638425678924 Кб, 620x406
#317 #353018
>>353012
На орбиту Солнца.
#318 #353025
>>353018
А почему орбиты Солнца тогда сейчас не запружены такими лунами?
#319 #353031
>>353012

>куда улетит Луна с орбиты Земли?


Никуда она не улетит, наркоман.
#321 #353039
>>353032
И что? Когда земля станет совершать оборот вокруг своей оси за месяц, луна полетит обратно.
#322 #353041
>>353025
Запружены. Планеты называются.
#323 #353062
теоретически, на нынешнем уровне технологий, если построить mass driver (рэилган) на луне для запуска космических аппаратов к другим планетам и в дальний космос, какую скорость можно получить для аппарата весом в несколько тонн? скажем в 10 тонн
#324 #353063
Аноны, нужна инфа по хзапускам для всякой хуиты. Проблема в том, что она должна быть точной.
Что нужно:
Параметры орбиты.
Точная масса на орбите (с остатками топлива).
Точная масса на старте.

Неужели более-менее годные цифры только для всяких Аполлонов есть?
#325 #353065
карбон не пропускает электромагнитное излучение, так? тоесть если обшить карбоном все заводы, производящие электронику да и всё остальное в принципе, то им будут не страшны любые солнечные вспышки и даже взорвавшаяся в 10 километрах ядерная бомба?
#326 #353067
>>353062
проблема не в технологиях, а в средствах, которые не выделены
#327 #353068
>>353065

> карбон не пропускает электромагнитное излучение, так?


нет
#328 #353073
>>353068
Как тогда можно все анально огородить от чертового излучения чтоб техногенная цивилизация не накрылась плазмой?
#329 #353074
>>353063

>Параметры орбиты


Открытая бд наса/норад

>Точная масса на старте


гугл

>Точная масса на орбите (с остатками топлива).


а вот это бес его знает, вряд ли в открытом доступе есть
#330 #353075
>>353073

У тебя и так пердак плазмой просто переливается, поздно уже задумываться об излучении, мои соболезнования (нет)
#331 #353078
>>353073
поглубже в землю закопайся и всё норм
#332 #353079
>>353078

Ахахах, ты даже не представляешь, насколько в тему прозвучал этот совет
#333 #353081
>>353073
Свинцовым экраном всю планету огораживай на момент вспышки и норм. Только так можно прямо совсем анально... Или сферу своей плазмы вокруг всей планеты намутить, только подальше а то своим же излучением все положим
#334 #353082
Что такое электрический разряд с силой в миллиарды ампер? Говорят такие в жидком ядре возле твердого ядра Земляшки происходят регулярно, что будет если один такой шарахнет в атмосфере вместо рядовой молнии, сильно пиздос будет?
#335 #353084
>>353081

Не выйдет, весь свинец скормили твоим клонам и тебе лично, прям в ебло начиняли
neil-blevins-megastructures-3-alderson-disk-4[1]171 Кб, 1024x650
#336 #353085
Сап, поясните дрочеру на мегаструктуры почему не в маня реальностях приклейтед не сможет оправдать ожиданий даже если его заспавнить тупо на месте на халяву. Короче как это все разнесет.
#337 #353086
>>353085

Изи же, там в кадре слева видно твой багет, конструкцию изначально было нерентабельно спавнить рядом с таким катаклизмом, а теперь её разнесёт к хуям.
#338 #353094
>>353062
У электромагнитных ускорителей ограничения на скорость нет в отличие от химических.
Если не страшно распидорасить луну ускорять ебовым ускорением и достаточно места/электричества/бабла/огурцов - можешь хоть до 0.99с разгонять.
#339 #353095
>>353082

>что будет если один такой шарахнет в атмосфере вместо рядовой молнии, сильно пиздос будет?


Нет, во время полярных сияний примерно такой ток и выходит. 10 тыщ раз молния ударит и ток пройдёт.
#340 #353096
>>353085
Непонятно, нахуя нужен диск Альдерсона если есть нету кольцо Нивена? Какие профиты у этой хрени?
sage #342 #353105
>>353101
Ты статью-то дальше заголовка читал? Сказано же
Now, it's always possible that there's something fishy that happened in the star's past that we can't know about today. It's possible that it was born as a higher-mass star and something stripped the outer layers off, reducing the star's lifetime precipitously. It's possible that the star absorbed some material later-in-life that changed its heavy element content, skewing our perceptions today. Or it's possible that we've got a misunderstanding in the subgiant phase of the stellar evolution of these old, low-metallicity stars. These unknowns (and in some cases, unknowables) are possible sources of errors when we try and compute the ages of the oldest stars.

Так что перестань покупаться на кликбейты. На самом деле проблемы есть, но не с возрастом Вселенной, а с пониманием механизмов происходящих в некоторых звездах. Звезда Пшибыльского например, или Табби звезда. А если звезды в шаровых скоплениях, то хуй вообще знает.
#343 #353110
>>353101
Не верно определили возраст. Либо проебали какой-то параметр, либо какая-то поебень произошла во время эволюции звезды, которая сейчас не определяется.
#344 #353138
>>353105

>Так что перестань покупаться на кликбейты. На самом деле проблемы есть, но не с возрастом Вселенной, а с пониманием механизмов происходящих в некоторых звездах. Звезда Пшибыльского например, или Табби звезда. А если звезды в шаровых скоплениях, то хуй вообще знает.



Ну ок, пусть так, и это погрешность методов измерения и некомпетентность как наблюдателя. Но все же анон? А что вдруг мы все же найдем такой объект\объекты? Ну скажем звездная система с карликовой звездой, или блуждающая планета которые будут реально старше чем наша вселенная, причем скажем раз в десять? Мы сможем это определить по "состоянию" элементарных частиц. В остальном же кроме возраста такие объекты будут иметь те же параметры и абсолютно также взаимодействовать с материей и энергией как всё остальное. Как то крипотно..... какие будут оправдания тогда??
#345 #353140
>>353138

>какие будут оправдания тогда?


Сначала проснись и оправдайся за свои обосранные штаны.
#346 #353143
>>353074
Я прост хочу узнать потери на аэродинамику и прочую хуйню. Идеальная дельта - выдроченные гравипотери = вся хуйня.
С LEM Аполлона получилось расхождение в 70м/с, что охуенно, если учитывать то, что наклонение и градиент гравитации не вставлял. Хочу вот что-то с атмосферой чекнуть.
#347 #353146
>>353140
Я другой анон.
#348 #353170
Аноны, объясните мне смысл дельты я про дельта-вэ. Вот что оно означает? Это как-то связано с тем, что в космосе траектория неразрывно связана со скоростью? Как я понимаю, невозможно двум телам/аппаратам двигаться по одной и той же траектории, имея каждый разную скорость в одной и той же точке траектории
#349 #353173
>>353170
Характеристическая скорость в википедии для любознательных. Предположение у тебя правильное.
#350 #353175
>>353173

>Предположение у тебя правильное.


Спасибо за подсказку. В википедию ещё не лез, потому что хочу понять, а не выучить, так что продолжаю цепь предположений: Тогда полагаю, что изменение траектории также почти тождественно связано с изменением скорости... Вот только для изменения траектории на одну и ту же может потребоваться менять скорость на разную величину, в зависимости от того, в какой точке мы её меняем, верно?
#351 #353176
>>353175

>менять скорость на разную величину, в зависимости от того, в какой точке мы её меняем, верно?


...стоп. Но ведь если менять скорость в разных точках, то и траектории получатся разные, верно? Тогда получается, что если нам нужна какая-то строго определённая траектория, то есть только одна точка нашей текущей орбиты, в которой мы можем врубить движки и изменить свою скорость на N м/с, чтобы получить как раз нужную орбиту. N - это и есть дельта?
#352 #353178
>>353176
Это дельта манёвра. Можно получить такое же изменение орбиты и в другой точке, но дельта там будет другой.
Олсо, дельта зависит от гравитации дохуя, потому вместо уи юзай скорость истечения (уи считается через g прост), если хочешь некую идеальную циферку получить, с которой потом всё сравнивать будешь. Учёт гравитации, кстати, в формуле с уи топорный, поэтому лучше идеальную хуячить, а потом через интеграл от градиента гравитации и вектора дилды брать, на вики формула есть.

Надеюсь не слишком запутанно пояснил.
#353 #353183
>>353178

> Это дельта манёвра.


А есть дельта не манёвра?

> Можно получить такое же изменение орбиты и в другой точке


Как? Можно пример? Просто, например, импульс в апоцентре сдвинет перицентр и наоборот, других способов их сдвинуть я не вижу; а всякие изменяющие эксцентриситет манёвры сильно меняют общую форму эллипса?

>но дельта там будет другой.


А когда говорят, условно скажем, "чтобы допиздовать из точки А в точку Ж на луну, нужно столько-то дельты", это какую дельту они имеют в виду и откуда они её взяли?

> Олсо, дельта зависит от гравитации дохуя, потому вместо уи юзай скорость истечения (уи считается через g прост), если хочешь некую идеальную циферку получить, с которой потом всё сравнивать будешь.


А что за уи?

>Учёт гравитации, кстати, в формуле с уи топорный, поэтому лучше идеальную хуячить, а потом через интеграл от градиента гравитации и вектора дилды брать, на вики формула есть.


В статье про характеристическую скорость? А у Левантовского это есть? блин, надо бы его в блокнот конспектировать, а не просто читать.

> Надеюсь не слишком запутанно пояснил.


Вроде не очень, благ тебе.
#354 #353184
>>353183

> Как? Можно пример? Просто, например, импульс в апоцентре сдвинет перицентр и наоборот, других способов их сдвинуть я не вижу; а всякие изменяющие эксцентриситет манёвры сильно меняют общую форму эллипса?


Я слишком сумбурно выразился; я хотел сказать, что не вижу разных способов менять определённую одну траекторию А на определённую другую траекторию Б, за исключением манёвров с промежуточными траекториями (когда между А и Б мы некоторое время болтаемся и маневрируем на траектории В, а может даже Г и Д ещё можно прикрутить), и за исключением манёвра поворота плоскости орбиты, где вариантов дохуя.
#355 #353185
Аноны, поясните за магнитное поле Ганимеда. Я так правильно понял, что оно создает лишь локальный пояс защиты у экватора, а на полюсах и высоких широтах оно наоборот фокусирует говно и там говно? Ну типа оно же все внутри магнитного поля Юпи, так что оно должно засасывать все заряженные частицы как пылесос и по дугам их на полюса отправлять?
Capture10 Кб, 747x110
#356 #353192
>>353183

>А есть дельта не манёвра?


Дельта всей ракеты, например.

>Как? Можно пример?


Например нужно зарыться в атмосферу Марса пролётной траектории. Можно пшикнуть дохуя до пролёта, а можно пердеть уже подлетая, и там разница в дельте будет идти на порядки.

>А когда говорят, условно скажем, "чтобы допиздовать из точки А в точку Ж на луну, нужно столько-то дельты", это какую дельту они имеют в виду и откуда они её взяли?


Там isp меняется же, вот от этого и пляшут. Вообще, ни разу не видел, чтобы писали идеальную дельту (т.е. когда ракета вне гравитационного поля летит). isp ака уи - удельный импульс, гугли ит.

>В статье про характеристическую скорость?


В русскоязычной - про формулу Циалковского.
Но я любитель дохуя, если бы тут был анон-физик, было бы лучше.
Пикрил - идеальная дельта до орбиты для взлетной ступени Аполлона 17, которая получилась бы, если бы он сжигал всё это топливо без гравитации и не маневрируя. И еще рассчёт гравипотерь. Циферки почти сходятся, остальное - всякие алайменты с плоскостью орбиты CSM, изменение гравитации с высотой и всё такое. Можешь посмотреть скорость LEM на орбите после взлёта, и сравнить с разностью идеальной дельты/гравипотерь.
#357 #353193
>>353192

>с пролетной траектории


>пшикнуть за дохуя до пролёта


Я спать кароч. Если накосячил в ответе и особенно с циферками - пофиксите.
#358 #353194
>>353192
Да, в рассчётах не максимальная дельта, а именно по тому топливу, которое он сожрал, пока летел до орбиты. Там в баках еще довольно дохуя оставалось. Поэтому не смотрите на то, что на Вики, смотрите на selected mission weights насовские.
#359 #353196
>>353192
И p[t] это угол вектора тяги к г g, меняется по времени. Точной инфы нет, поэтому функция задается кусочно.
#360 #353203
>>353170

>Вот что оно означает?


Запас скорости. На сколько метров в секунду при актуальном количестве топлива/движков/массы можно будет поменять скорость аппарата.
#361 #353225
>>353223
Ты поменьше дрочи и побольше читай.
https://ru.m.wikipedia.org/wiki/Марсианский_метеорит
#362 #353246
>>353096

>Какие профиты у этой хрени?


ну типа много больше площади, гравитация без вращения и можно с двух сторон аутировать.
#363 #353261
>>353246

>гравитация без вращения и можно с двух сторон аутировать.


Охуительные истории.
#364 #353289
>>353261

>Охуительные истории.


Ещё какие... Маняцивилизацич могущая сделать ЭТО должна штамповать сотни тысяч колец Нивена ибо требуемое количество материала для колец в десятки тысяч раз меньше. Тем не менее эти истории не я сочинил, диск реально должен будет иметь собственное гравитационное поле: но у меня закладываются сомнения по поводу его равномерности кроме центра диска, чем ближе к краю тем больше будет угол по типу этой модели:

https://en.m.wikipedia.org/wiki/Alderson_disk

http://www.pvsm.ru/budushhee-zdes/163963
Это дальнейшее развитие идеи мира-кольца: речь идёт уже о создании искусственного диска вокруг звезды, толщиной несколько тысяч километров. Автором идеи является Дэниел Алдерсон, участвовавший в написании программного обеспечения зондов «Вояджер-1» и «Вояджер-2». Применительно к Солнечной системе, внешний край диска должен проходить между Марсом и Юпитером.

По периметру внутренней границы диска, обращённой к звезде, проходит высокая стена, частично защищающая от губительного излучения. Обитаемая зона диска находится в… зоне обитаемости. Как и в случае с планетами, внутренняя и внешняя части диска непригодны для существования людей — у звезды слишком жарко, у внешнего края слишком холодно. Хотя технические достижения людей будущего, способных создать ТАКОЕ, вполне могут сделать обитаемыми и менее благоприятные области диска. При этом жить можно будет на обеих поверхностях диска. При этом он будет создавать достаточную силу гравитации, направленную перпендикулярно к его плоскости, чтобы люди смогли комфортно жить.

Согласно грубым расчётам, масса такого диска превзойдёт массу Солнца. Более того, требуемый объём материала многократно превышает доступное в Солнечной системе количество. Правда, из обычных камней такой диск построить не получится, для этого не хватит прочности. Так что строительство такого сооружения потребует создания совершенно новых материалов огромной прочности, и их производство даже не в планетарных, а в «системных» масштабах.

Диск Алдерсона редко упоминается в литературе. Например, в комиксах Ultraverse и повести Missile Gap Чарльза Стросса.
#364 #353289
>>353261

>Охуительные истории.


Ещё какие... Маняцивилизацич могущая сделать ЭТО должна штамповать сотни тысяч колец Нивена ибо требуемое количество материала для колец в десятки тысяч раз меньше. Тем не менее эти истории не я сочинил, диск реально должен будет иметь собственное гравитационное поле: но у меня закладываются сомнения по поводу его равномерности кроме центра диска, чем ближе к краю тем больше будет угол по типу этой модели:

https://en.m.wikipedia.org/wiki/Alderson_disk

http://www.pvsm.ru/budushhee-zdes/163963
Это дальнейшее развитие идеи мира-кольца: речь идёт уже о создании искусственного диска вокруг звезды, толщиной несколько тысяч километров. Автором идеи является Дэниел Алдерсон, участвовавший в написании программного обеспечения зондов «Вояджер-1» и «Вояджер-2». Применительно к Солнечной системе, внешний край диска должен проходить между Марсом и Юпитером.

По периметру внутренней границы диска, обращённой к звезде, проходит высокая стена, частично защищающая от губительного излучения. Обитаемая зона диска находится в… зоне обитаемости. Как и в случае с планетами, внутренняя и внешняя части диска непригодны для существования людей — у звезды слишком жарко, у внешнего края слишком холодно. Хотя технические достижения людей будущего, способных создать ТАКОЕ, вполне могут сделать обитаемыми и менее благоприятные области диска. При этом жить можно будет на обеих поверхностях диска. При этом он будет создавать достаточную силу гравитации, направленную перпендикулярно к его плоскости, чтобы люди смогли комфортно жить.

Согласно грубым расчётам, масса такого диска превзойдёт массу Солнца. Более того, требуемый объём материала многократно превышает доступное в Солнечной системе количество. Правда, из обычных камней такой диск построить не получится, для этого не хватит прочности. Так что строительство такого сооружения потребует создания совершенно новых материалов огромной прочности, и их производство даже не в планетарных, а в «системных» масштабах.

Диск Алдерсона редко упоминается в литературе. Например, в комиксах Ultraverse и повести Missile Gap Чарльза Стросса.
#365 #353292
>>353289

>этой модели:


http://www.tp4.rub.de/~jk/science/gravity/chapt_alderson.html
Вообще ожидал материал по проще, но так даже лучше.

Но кому не понятно: По центральной круговой линии диска на равном расстоянии от внутреннего и внешнего края будет под прямым углом прижимать, но на внешнем или внутреннем углу будет под 90 градусов угол, как будто на отвесной скале "стоишь" реально стоять можно будет только на боку диска. На самом деле все распидарасит без оче йоба материалов
#366 #353309
Антоша,выручай. В школе заставляют делать выпускной проjэкт. что то типо мини исследовательской работы
Выбрал астрономию. Тут и до сдачи недалеко. А я даже не определился с темой!
Реквест : нужна увлекательная тема с хоть какой нибудь практической частью
#367 #353310
>>353309
Посчитай скорость движения Луны по орбите, использую визуальные наблюдения.
#368 #353313
во сколько раз плутон меньше луны?
#369 #353317
>>353310
можно поподробнее ?
#370 #353318
>>353313
В 1.3 раза примерно.
#371 #353319
>>353203

>На сколько метров в секунду при актуальном количестве топлива/движков/массы можно будет поменять скорость аппарата.


Я так понимаю, максимально теоретически возможная разница скоростей при заданных условиях?
#372 #353320
>>353309
Про Кассини расскажи.
#373 #353322
>>353320
а ещё лучше накати за него
#374 #353323
>>353309

>нужна увлекательная тема с хоть какой нибудь практической частью


Вывод законов Кеплера из второго закона Ньютона и применение его к теоретическим расчётам параметров орбиты Урана.
#375 #353328
Кеплеры, а можете наглядно пояснить, что такое средняя/истинная аномалия как параметр орбиты? Все остальные пять параметров понял, а это нет.
#376 #353367
>>353319
Задолбал. Летит аппарат по солнечной орбите, 1км/с, дельтаве 2км/с. Если выдаст всю дельту мгновенно — будет 3км/с, топлива в баках не останется. ВСЁ.
#377 #353371
>>353317
НАБЛЮДАЕШЬ ЗА ПОЛОЖЕНИЕМ ЛУНЫ
@
ЗАПИСЫВАЕШЬ НАБЛЮДЕНИЯ
@
ИСПОЛЬЗУЕШЬ МАТАН
@
ПРИБЫЛЬ
#378 #353374
>>353367

>Летит аппарат по солнечной орбите, 1км/с, дельтаве 2км/с. Если выдаст всю дельту мгновенно — будет 3км/с, топлива в баках не останется.


(Для простоты допустим, что речь про 2д мир): но если он выдаст дельту перпендикулярно вектору скорости, то у него скорость не станет 3 км/с.

>Задолбал


Хорошо, больше не достаю. Мне стало в целом гораздо понятнее, спасибо, анон(ы)
#379 #353398
>>353292

Ну и ретард
#380 #353422
>>353374

>(Для простоты допустим, что речь про 2д мир): но если он выдаст дельту перпендикулярно вектору скорости, то у него скорость не станет 3 км/с.


А если даст в обратном направлении, то вообще станет 1 км/с. Если перпендикулярно - то складываешь текущий вектор скорости с новым вектором скорости. Текущий был 1км/с в одном направлении
Новый вектор будет в другом направлении и (сюрприз) 2км/с.
#381 #353481
Почему ракеты не запускают с гор, с того же Тибета? Ведь чем выше от уровня моря, тем меньше атмосферы нужно преодолеть.
Железная дорога в Тибет есть вроде.
#382 #353483
>>353481
Платина.
Можно возрождать шапку которую никто не читает
Космос это не как высоко, а как быстро. На фоне сэкономленных 200 м/с дельты ты теряешь простоту обслуживания стартовой площадки и доставки ракеты. Полнейший проеб по логистике. Больше профита с моря на экваторе пускать.
#383 #353486
>>353483
Спасибо, добрый анон! Про экватор не знал но догадывался.
#384 #353512
>>353483
Насколько проигрывают жыды, запуская ракеты в космос против вращения земли?
#385 #353513
>>353512
Почти столько же, сколько и все остальные державы запускающие спутники на полярку.
#386 #353535
>>353512
А что им мешает запускать по направлению вращения? Религия?
#387 #353542
>>353535
Районы падения ступеней оказываются не там, где им выгодно.
#388 #353568
Что будет, если взорвать ядерную бомбу на Титане? Как поведет себя его атмосфера? Она не загорится? Ударная волна будет мощнее? Гриб будет выше из-за низкой гравитации?
photo2017-09-0917-48-11123 Кб, 1280x853
#389 #353570
>>353568
Как она загорится? Там же окислителя нет. Кроме того в нижних слоях атмосфера на титане почти 100% азот.
#390 #353576
>>353568

>Она не загорится?


Нет

>Ударная волна будет мощнее?


Да.
Гриб будет выше из-за низкой гравитации?
Да.
i11 Кб, 263x215
#391 #353582
Объясните мне почему мы так сильно полагаемся на образцы грунта с Луны и на марсианские метеориты при составлении геологической истории этих тел? Разве один камень может нам что-то сказать? Вот если бы у вас в руках был один камень с Земли из одного региона, вы бы смогли восстановить всю геологию планеты по одному камню из одной точки? А в случае с Луной и Марсом ученые пытаются.
6654173 Кб, 650x487
#392 #353583
>>353568
Ты мне напомнил, как я в детстве в интернете этом вашем прочитал, что учёные отправили какой-то отработанный спутник в центр какой-то планеты, из-за чего она может загореться, стать ярче солнца и спалить землю.
Так испугался помню, думал скоро помрём.
А ещё наложил кирпичей с пикрелейтеда, когда увидел.
Я уже и забыл об этом всём, а ты мне напомнил.
#393 #353586
>>353582
Потому что изотопный состав разный. Больше интересует не сами элементы, а проценты изотопов в образцах, в том числе заключенных в породах газов. Они в среднем по астрономическому телу везде одинаковые, а от тела к телу - разные.
#394 #353592
>>353583
Поорал с предсказаний.
А прочитал ты тогда про Галилео, его об Юпитер убили, почти как совсем скоро Кассини
#395 #353611
Как космонавты не замерзают и не сгорают на Луне, где температура и -200 до +100?
#396 #353615
>>353611
А как они не делают этого в открытом космосе?
#397 #353616
>>353513
А те сколько проигрывают?
#398 #353617
>>353592
Помянем!
#399 #353618
>>353615
В космосе нет атмосферы, но вопрос тоже неплохой.
#400 #353620
>>353615
У космонавтов\астронавтов есть йоба-скафандры с системой жезнеобеспечения. Это такая переносная индивидуальная сычевальня, с подачей воздуха, охлаждением\подогревом. Только пека нет и бабушка пирожки не носит.
#401 #353624
>>353620
И не покапчевать двапчу
#402 #353632
>>353620

>нет пеки и пирожков


МУСК ПРИДИ ПОРЯДОК НАВЕДИ
#403 #353636
Что бы изменилось на Земле, будь у нас вместо Луны Меркурий или вообще Ганимед?
А) На месте Луны
Б) На таком расстоянии, чтобы куда более большой Ганимед/Меркурий выглядели бы с Земли как Луна сейчас.
#404 #353639
>>353632

>МУСК ПРИДИ ПОРЯДОК НАВЕДИ


ТЕРМОДИНАМИКА ВПЕРДЕ! ЭНТРОПИЯ ПРИЙДЕ ПОРЯДОК НАПЕРДЕ! РЕЦЕССИЯ СТАГНАЦИЯ И ХАОСА КАСТРАЦИЯ, РАВЕНСТВО И РАВНОВЕСИЕ ГРЯДЕ!
#405 #353679
>>352045 (OP)
Бля, сегодня прочитал, что Солнце раздуется и ебанет. Как скоро это будет? А то охерел от того, что температура после вщрыва Солнца через 8 минут на Земле достигнет 3-5 тысяч градусов. Это ж не спастись нигде?
Вот где прочитал:
http://www.mk.ru/science/2017/09/14/uchenye-predskazali-skoruyu-gibel-solnca-i-zemli-posle-anomalnykh-vspyshek.html
Так взорвётся или нет? Спасибо.
#406 #353680
>>353679
охуительные истории
ты попал по адресу
такого вскоре не будет
#407 #353682
>>353679
Че то как то поясняют смоченной желтой мочей вилкой, хоть бы для приличия про сверхновую не вякали, у нас тут вроде звезды не могут внезапный Бетельгейзий испытывать от того что они карланы с массой в одну солнечную и рваться сверхновыми от того что у них НИБАМБИТ.
#408 #353683
>>353679
Оно просто раздуется, но не ебнет. Тоби пизда все равно
planetballs-countryballs-Комиксы-длиннопост-20361612,4 Мб, 495x9714
#409 #353684
>>353679
Напомнило:
#410 #353691
>>353679
Звезды солнечной массы не взрываются. Не читай желтухи.
#411 #353692
>>353691
>>353684
>>353682
>>353680
Так ёбнет или нет? Или ваши комментарии в стиле "не ёбнет, но вспышками зажарит"? Я умирать не хочу.
#412 #353693
>>353683
По ссылке ведь тоже учёные приводятся. Они ошибаются?
#413 #353696
>>353693
Я офк не эксперт, но судя по прошедшим миллиардам лет, жизнь полностью не снесет. Но магнитная буря может быть настолько охуевшей, что нас отбросит в 19 век
#414 #353705
>>353620
Это был наводящий вопрос для этого >>353611
#415 #353709
>>353696
Каким образом, можно поподробнее? Неужели снова нельзя будет электричество включить?
#416 #353710
>>353709
Ты про ЭМИ слышал от ядерного взрыва? Вот представь, что помимо облучения мы получим ЭМИ на всю планету. В 1859 фонил и искрил каждый кусок железа, ебать, тут никакая заглушка не поможет. Просто пизданёт всё.
#417 #353714
Что за хуйня лед 7?
#418 #353716
Если свет есть частица, почему её нельзя поймать? Наловить себе частиц света, кинуть в банку и ходить без фонарика ночью?
#419 #353718
>>353636
Приливы посильнее и луна поярче.
Может даже о колонизации поговаривали бы.
#421 #353727
>>353692
Оно раздуется и может быть поглотит Землю, а может нет. На этот счет есть разные мнения, но жить на Земле будет невозможно через миллиард-полтора лет. Ты к тому времени будешь давно дохлый.
Звезды солнечной массы не взрываются, они распухают, аж пиздец, потом скидывают внешнюю оболочку и остается небольшой шарик, из кислорода/углерода, размером с Землю, но очень горячий. Прям пиздец какой горячий.
#422 #353740
>>353716
Ты воду дохуя в шапку вязаную наловить можешь?
#423 #353742
>>353740
Килограмм пять без проблем.
#424 #353849
>>352944

>бактерии зародились пару миллионов лет назад.


Заебись динозаврикам было - руки перед едой не мыли.
#425 #353864
>>353849
Двачую

>>352944

>Чет ты пиздишь


>Не разбираюсь


>Анус ставлю


Ну вот ты и обосрался, анончик
#426 #353865
>>353864
Чет с этого поста меня проперло на максишизу:
ОБОСРАЛСЯ, МАЛАФЬИ НАСАСАЛСЯ, ГОВНА НАЖРАЛСЯ, С ПОДЛИВОЙ ОБДРЫСТАЛСЯ, ПЛАЗМОЙ ПРОПЕРДЕЛСЯ, ГОВНА НАЕЛСЯ, МАГМОЙ ОБОССАЛСЯ, В ГОВНЕ ОКАЗАЛСЯ, ВОНИЩЕЙ ПРОВОНЯЛСЯ
мимо рандомная кучка произвольного говна
#427 #353866
>>353865
говно
#428 #353868
>>352045 (OP)

>#НОВОСТИ


#ВМИРЕ
#ПРОИСШЕСТВИЯ
#США
#УКРАИНА
#РОССИЯ
#САНКЦИИ
#КНДР
Учёные обнаружили место, где человек сможет стать бессмертным
Антон Милёшин 17.09.2017 02:56

>К сожалению, для того чтобы добраться до этого места, придётся покинуть нашу планету.


Астрофизики, которые занимаются изучением чёрных дыр, предположили, что если человек окажется рядом с этой космической аномалией, то он перестанет стареть. Об этом сообщает BBC.

>Учёные объясняют это тем, что вокруг чёрных дыр из-за их огромной массы происходит искажение пространства и времени. Поэтому чем ближе человек будет находиться к этому космическому объекту, тем медленнее для него будет проходить время. Таким образом, пока на земле пролетит несколько миллионов лет, для человека около чёрной дыры пройдёт несколько секунд.


https://life.ru/t/новости/1044197/uchionyie_obnaruzhili_miesto_ghdie_chieloviek_smozhiet_stat_biessmiertnym/amp
гептил59bd095f370f2cc4308b4570288 Кб, 1500x1000
#429 #353869
а у Пухлика гептил в ракете? Как они там сидят в этой машине?
#430 #353870
>>353869
Я, опять же, не разбираюсь, но вроде бы гептил не может храниться годами в ракете. А для военных ракет это важно. Один раз заправил и держишь годами на готове.
#431 #353873
>>353870
нет, наоборот гептил и хранится. Это сжиженный кислород быстро прогревается.

Короче по цвету выхлопа можно определить?
Выхлоп опасен для персонала?
#432 #353874
>>353865

>мимо рандомная кучка произвольного говна


Собственно как и высеры не сведущих в определенной области, но громко кукарекающих о предметах ее изучения ананасов ;), так что все чики-мони, друг, твой пост даже больше в тему
#433 #353876
>>353869
Машина и водитель одноразовые.
#434 #353883
>>353873
После полетов Протонов никто дезактивацию территории не проводит, если там какие-то продукты горения и опасны или остались какие-то пары непрореагировавшего гептила, то в такой мизерной концентрации, что не опасно. Тем более под открытым небом.

Топливную смесь по цвету выхлопа определить можно, может анон даже поделится шпаргалкой итт
#435 #353885
>>353883
а хули цвет такой рыжий? это йод в составе или азотные окислы?
#436 #353886
>>353679

>приведет к тому, что нашу опустошенную, обгоревшую и оплавленную планету выбьет


Не так прочитал сами знаете какое слово и почти возбудился. Есть прон про то как Солнце имеет Землю во все щели?
#437 #353888
>>353886 -> >>352902
А прон... вроде чет было..
#440 #353892
>>353886
Кассини вон Сатурн отшатал.
applause pickard391 Кб, 250x189
#441 #353893
>>353868
Какой-то ученый изнасиловал журналиста прочитал про релятивистское замедление времени?
#442 #353894
>>353869
>>353885
Рыжие азотные окислы. Если тетраоксид - ещё ничего, эти могут вообще азотную кислоту ебануть ради великой идеи чукчи или как там их.
АК - лютейший пиздос, наша ебнувшая ракета на старте эпохи забрала жизни сотни людей, в т.ч. даже генерала.
Но да, скорей всего топливо НДМГ, другие топливные пары так не выглядят (если они новую не изобрели, лол)
#443 #353895
>>353894
Не генерала а маршала
q-whatjpg[1]22 Кб, 550x250
#444 #353896
>>353868
>>353893
Не ну фактически он почти прав.. Только проматывать время находясь вплотную к горизонту черной дыры там уже будет не человек, а плазма с излучением.
#445 #353898
>>353896
Так он в скафандре будет
#446 #353900
>>353896
У сверхмассивной уцелеет же, если в аккреционный диск не влетит
#447 #353903
>>353898
Попробуй в скафандре подставиться под поток частиц в БАКе, а ведь космонавту на околосветовой скорости придется вращаться вокруг горизонта событий, что бы не завалиться за него и любая частица что в него врежется окажет на него воздействие не слабее чем, если бы он подставился бы под поток из ускорителя. А возле чд обычно полно всякого хлама.

>>353900
Да уцелеет, но аккреционный диск тоже проблема очень серьёзного характера, даже если там будет самую малость частиц по сравнению с типичным аккреционным диском. Это надо искать голую сверхмассивную чд вышвырную из галактик и доевшую все оставшиеся говна вокруг и все равно 100
% ой гарантии не получить дофига миллизивертов не будет.
#448 #353906
И ещё, мы вон не можем заставить исследовательский зонд, принять нормальную круговую орбиту вокруг Юпитера в стиле МКС, а ведь надо всего лишь сбросить жалкий десяток километров в секунду, как будем выходить на кольцевую вокруг сверхмассивной чд, гася десятки тысяч км в секунду, чтобы нормально проматывать время?
#449 #353907
>>353896
Возле СМЧД спагеттификации нет.

>>353903
Так это ж просто прочитавший статью на вики журналист. Какие зиверты, какая акреция? Ты еще про ту же спагеттификацию вспомни или про релятивистское искажение длины.
#450 #353908
>>353906
Маняэмдрайвом, естественно
#451 #353909
>>353906
Да никак, епту. Чтобы нормально время мотать надо быть сравнимо близко к горизонту. Расстояния порядка 1.1 горизонта и ближе.
А на самом деле тебя засосет не только на фотонной сфере, ты сможешь быть на орбите только на 3 радиусах шварцнеггера.
#452 #353911
>>353909
А чому ближе нельзя?
Nigga+nigger+niggest+this+five+word+rule+is+silly11dc1c3668[...]27 Кб, 640x257
#453 #353912
>>353911
На двух радиусах Штугернбрауера у тебя уже нет стабильной орбиты, тебя либо унесет в открытый космос с йоба-скоростью, либо засосет в горизонт. Метрика пространства-времени люто искажена.
На полутора радиусах Шиндлера так у тебя вообще на орбите могут быть лишь фотоны. И то не могут, т.к. малейшая флуктуация - и их либо в дырень, либо в пиздень наружу.
Такие пирожки с котятами - ты их ешь, они пищат. Епту. Ты эт спрашивай еще если интерес, я поясню без бэ.
#454 #353913
>>353911
Искажает пространство там по сильнее чем время, его как бы.. скручивает, из - за этого даже если например лететь в противоположную вращению чд сторону то расстояния на котором она может тебя поглотить значительно так увеличится, но даже если вдоль вращения, все равно эти колебания неслабо так сместить могут и в один прекрасный момент.. опа и поглотило..
#455 #353914
>>353912
А чё ещё спрашивать-то? Ты все объяснил
#456 #353915
>>353914
Да я все про космос знаю кароч ептубля. Про сатурны-шептуны без бэ поясню нах.

>>353913
Это мы с тобой, братуха, кароч еще только про черные дыры Штакеншнайдера говорим, а про Кековские еще не говорили. Для пацанов поясню - про статичные незаряженные. С крутящимися вообще пиздос, там эргосферы есть. А если начать тереть за кратные дыры, там ядрен матан пойдет бля.
#457 #353917
>>353915
Так и я тоже не мало, лол. В этих ваших дырках не разбираюсь только, ибо после пейсэнджина боюсь их
#458 #353918
>>353915
Мне иногда кажется что если бы объекты реальности были бы достаточно умными что бы уметь в матан и пытались бы осознать то насколько сложно описать их собственные флуктуации, то все бы по пизде пошло кароч... А так они не знают ничего о себе и им норм, сидят заворачивают тензоры в момент столкновения двух вращающихся чд и гравитационными волнами поплевывают и ничего не лагает...
#459 #353919
>>353918
а когда пытаешься просимулировать этож бля все коптит лагает, тормозит, одну секунду рендерит по пол часа, бесит пздц..
#460 #353920
БОЛЬ ПОЗНАНИЯ, СТРАДАНИЯ ПОНИМАНИЯ! ОЩУТИ ИХ! ВПИТАЙ ИХ! ЖИВИ ИМИ!
#461 #353922
>>353920
Не надо мне тут вилкой за страдания пояснять, быть тупым дауном и веровать в пиздец погоняющий пиздецом оперирующий невнятным пиздосом МНОГОКРАТНО куда больше страданий приносит, так что не надо мне тут я нормально..
#462 #353923
>>353912
Маняфантазии какие-то. Как и ЧД впрочем.
#463 #353924
>>353922
аутирую..

>>353923
Это ты маняфантазия, а чд вполне себе плюются плазмой, если аккреционный диск слишком насыщен контентом, и эту плазму волне детектят с помощью нормального оборудования не могущего всё приукрасить субъективными галлюцинациями.
#464 #353925
>>353923

>пук

image433 Кб, 600x450
#465 #353927
>>353923
Ты ща за базар отвечать будешь, по`ел?
Ебать, ЧД у него маняфантазии. Метрика Керра у него хуйня, блядь.
Ты еще скажи, нах, что у тебя есть лучшее объяснение сверхмассивного компактного радиоисточника в Стрельце А, ёпту бля.
#466 #353928
>>353912
Тащемта ты несешь хуйню, аккреционные диски это доказывают. В них вещество гуляет по вполне стабильным круговым орбитам и сходит на горизонт только в результате турбулентных неоднородностей и трения между частицами диска.
#467 #353930
>>353928
Цифры давай епту. На каком расстоянии аккреционные диски от ГС идут.
#468 #353932
>>353925

>срак

#469 #353937
>>353924
Это кто-то видел?
#470 #353938
>>353927
Может инопланетный радиомаяк
#471 #353939
>>353923
Тут ещё принято считать безо всяких оснований что видимые звезду, такие-же солнца как наше. хуле ты от них хотел
#472 #353941
>>353938
В каждой галактике, ага
#473 #353943
>>353939
Звезды видно а ЧД нет
>>353941
Как что-то невозможное
#474 #353948
>>353943
Ага, устроили себе межгалактический интеренет с пингом в пару сотен миллионов лет
#475 #353949
>>353948

> маяк


> интернет


Пиздюк слепошарый
#476 #353950
>>353949
Бля, я подумал что ты имел ввиду использование этих маяков для связи. Юзать их как собсна маяки ещё более долбоебская идея
#477 #353951
>>353950
Могут для чего угодно. Собсна никто не знает что это.
#478 #353962
>>353951

>Собсна никто не знает что это.


якубович_даладна.пнг
pepe thinking29 Кб, 640x519
#479 #353976
>>353939

>Тут ещё принято считать безо всяких оснований что видимые звезду, такие-же солнца как наше. хуле ты от них хотел


По-моему, перетолстил.
#480 #353984
>>353894
но окислы ведь образуются в произвольных пропорциях? Припоминаю что там какие-то очень вредные есть и на химических предприятиях их отдельно измеряют. Они там тонкой струйкой круглые сутки стравливаются.
#481 #353999
>>353976

>По-моему, перетолстил.


Пффф, ты эту одномерную струну называешь толстой?! Ща напишу тру толстоту:
Ещё тут принято считать без всяких оснований, что у нас в ядре земли один лям атмосфер давления.
Ещё тут принято считать, безо всяких оснований, что во вселенной вообще есть какая то там энергия и материя, да ещё и пространство ох тыж бля...
Ещё тут принято считать, безо всяких оснований, что наша реальность действительно на самом деле существует.
Ещё тут принято считать, безо всяких оснований, что нейтронные звезды состоят именно из нейтронов.
Ещё тут принято считать, безо всяких оснований, что в космосе прямо до жопы радиации.
Ещё тут принято считать, безо всяких оснований что нигде во вселенной больше не осталось антивещества.
Ещё тут принято считать, безо всяких оснований, что именно пустота в своей высшей форме йобнула большим взрывом.

Я кончил и готов к подаче в ебло шариков ненависти и говна размером с Бетельгейзе. А стоп я же не кукарекал а тупо примеры идеальной толстоты привез, так что... впрочем насрать
image85 Кб, 267x189
#482 #354033
>>353999
Перебор т.к. основания есть.
#483 #354051
>>354033
Естесссственно..
#484 #354052
>>353999
А уж сколько в СЕТИ принято считать разных положений, без всяких на то оснований)))0))
а смотри ж ты, курсы при универах проводят
https://www.youtube.com/watch?v=D28iGN4yT5w
#485 #354059
Вот у нас есть некий объедк из очень глубокого космоса вроде галактик и т.п. который при нормальных условиях разглядеть невозможно. Но вот совпадение, на пути луча от него к нам появляется достаточно массивный объедк вроде скопления или смчд, тем самым приближая визуально объедк позволяя видеть больше деталей вроде отдельных звёзд, пускай и немного искажённые линзой.
Есть ли примеры такого наблюдения или это удел будущих технологий, если это в принципе возможно?
#486 #354060
>>354059
Есть
#487 #354061
>>353999

>что в космосе прямо до жопы радиации.


А кстати реально почему в космосе прямо до жопы радиации, что так может фонить то блжад?! Не ужели было ПРЯМО ТААААК много сверхновых, гамма всплесков, джетов нейтронок, черных дыр и сверхмассивных черных дыр, что бы засрать весь космос этими долбаными микрозивертами? Он же уже должен был достаточно раздуться, да и энтропия же и все дела, звездо образование сильно упало.. бесит блжад! ЧТО ТАК НАФОНИЛО?! И реально что ли даже если вылетишь в меж галактический войд то сдохнешь от радиации если предположить что другие потребности в жизнедеятельности удовлетворяются Сколько ещё миллиардов лет нужно ждать что бы было умеренное количество радиации?
#488 #354062
>>354061
Опасные уровни радиации только, кажется, возле звезд. В межзвездном, а тем более в межгалактическом космосе более менее. Только есть рядом сверхновая не ебанет или гамма всплеск не прилетит
#489 #354063
>>354061
>>354062
Или не будет сверхжирной дырчатой чорки со здоровенным и плотным аккреционным диском.
148258953573 Кб, 640x363
Время супиртупого вопроса #490 #354064
Что будет если взять черную дыру и разогнать её до скорости 99.9999999999999999999999999999999999999999999999999999999999999999999999999999999999999999999999999999999999999999999999999999999999999999 световой? Насколько ситуация будет экстремально релятивистской возле её горизонта теперь? Что если взять три чёрные дыры заставить их почти почти столкнуться и потом их разогнать до вышеуказанной скорости, что за пиздец увидит сторонний наблюдатель? А что будет если все это поместить в ситуацию когда случается Большой разрыв\фазовый распад вакуума\Большое сжатие\случается тепловая смерть и они в процессе полёта испаряются Хокингами?

Напоминаю все ситуации ну более или менее физически допустимы и могут потенциально возникнуть в реальности хоть и с астрономически малой вероятностью.
#491 #354065
>>354064
Ща совсем мозг сломаю, а что будет если к причисленным ситуациям добавить встречную такую же одну\три черных дыры летящих на такой же скорости и должных столкнуться с первыми в процессе полета? А если не в лоб, а в бок да ещё и все вращаются примерно с такой же скоростью? Ой чувствую это ничто не способно будет рассчитать даже за сотню ближайших лет
#492 #354066
>>354064
>>354065
Ну ты написал.. аш слюнки потекли, жалко тут ничто не способно это затестить не на симуляциях, не тем более в реальности..

Аш самому интересно стало, но на всякий случай перенес вопрос сюда: http://www.astronomy.ru/forum/index.php/topic,57615.msg4161078.html#msg4161078 а то не сильно уверен, что тут его нормально прокомментируют.
#493 #354067
>>354064
Если горизонты соприкоснутся, то будет только 1 ЧД с суммой векторов движения, + какие-то ебалы вроде взаимной гравитации, - масса тратящаяся на излучение в частности гравитационное. Это в итоге, но в процессе слияния там образуется топология П-В достаточно ебалистическая, и где-то даже говорится что можно проложить в том аду путешествующее в прошлое траекторию.
Если же просто будет лететь одиночная ЧД на большой скорости, то применяя принцип относительности это будет просто дыра летящая на скорости. Даже ГС скорее всего не будет искревлён. И по факту это не ЧД врезается в землю, а мирно стоящая на орбите стремительно приближается и падает на ЧД.
Если добавлять количество, то кардинально ничего не поменяется скорее всего.
Сторонний наблюдатель это не тоже что и на самом деле. Мы сторонне наблюдаем фильмы, но это же не значит, что трансформеры живут на самом деле, это всего лишь фотоны. В то же время про фотоны относительно чд можно сказать одно, их засасывает и ни о каком стороннем наблюдении чд не в контексте научпопа речи быть не может. Если ты понимаешь о чём я говорю.
Всякие не стандартные условия не являются чем-то известным или доказанным. В принципе хоть сейчас можно придумать вселенную без гравитации и математически её прощитывать, но от этого научной фантастикой это быть не перестаёт. Прямых наблюдений нет, даже косвенных нет.

Ещё раз кратко.
1.Принцип относительность.
2. Добавление новых объедков в механизм слияния 2х ЧД ничего нового не даст. Хоть и топология будет максимум искажена. Если хочешь более упрощённо ,совсем упрощённо, посмотри на магнитные поля двух притягивающих обедков.
3. Бритва окама. "А что если ЧД попадёт в мангу по рубакам?" Все эти гипотезы о будущем земли хоть и имеют под собой основу не являются чем то материальным и не могут быть подробнее, чем ты и так знаешь.
2.1. Добавление углов или прочих хуйностей в механизм слияния 2х ЧД тоже не принесёт нового это тоже будет одна сферическая ЧД.
#494 #354069
Столько фиксов надо сделать. Но я думаю все тут сообразительные и поймут, где какие существительные надо доставить и что будущее не земли а вселенной.
>>354067
#495 #354071
lol
#496 #354073
>>354067

>Если ты понимаешь о чём я говорю.


Ну я знаю что ну никак не получится посмотреть под горизонт со стороны. А так же знаю что на них в принципе посмотреть сейчас крайне проблематично.

Если имелся один из этих двух смыслов то я понял.
#497 #354077
>>352365
Вселенная бесконечна в том плане, что у нее нигде нет конца, предела, стенки или подобной хуйни. Вот бесконечен ли ее объем -- неизвестно. Наблюдаемая вселенная имеет конечный объем.
#498 #354080
>>353727

>Ты к тому времени будешь давно дохлый.


За себя говори.
#499 #354081
>>354080
Манька собралась жить миллиард лет?
blob200 Кб, 401x604
#500 #354082
>>352045 (OP)
Подскажите пожалуйста, какая звезда, самая яркая во вселенной? Мне бы ее название.
#501 #354083
Кто и как определяет, что Млечный путь и Андромеда сближаются?
Может они наоборот отдаляются.
-zT7myyQ0SE35 Кб, 700x500
#502 #354084
Взгляни на это видео.
https://www.youtube.com/watch?v=GoW8Tf7hTGA
Можешь ли ты представить себе, как этот видос плачет и набирает дрожащими пальцами телефон звезду уровня MASS J0523-1403 или чутка побольше? А можешь ли ты представить планету HD 100546 b?
А теперь два вопрос: как может образоваться такая солнечная система, в которой такая звезда и такая планета, которая значительно больше звезды? Будет ли в таком случае звезда вращаться вокруг планеты?
sage #503 #354085
>>354084

>Будет ли в таком случае звезда вращаться вокруг планеты?


Пиздуй отсюда нахуй
#504 #354086
>>354085
Хуй соси, долбоёб. Это тред тупых вопросов, а не твой личный тред, откуда ты можешь выгонять любого неугодного.
#505 #354087
>>353999

>у нас в ядре земли один лям атмосфер давления


Если так, то почему его не распидарашивает?

>во вселенной вообще есть какая то там энергия и материя,


>наша реальность действительно на самом деле существует.


Что такое "наша реальность"? Это вообще более философия

>нейтронные звезды состоят именно из нейтронов.


Никто не знает, как там. Про звёзды вообще ничего неизвестно, что они светят. Просто Коперник спизданул и понеслось.

>в космосе прямо до жопы радиации.


Взяли и измерили
>>353999

>нигде во вселенной больше не осталось антивещества


Кто сказал?

>именно пустота в своей высшей форме йобнула большим взрывом.


Щито, бля? Пустота не может ёбнуть.
Хуйня твои вопросы
#506 #354088
>>354086
Там от массы а не от размера зависит. Скорее всего та звезда сжата оче сильно своей гравитацией и на поверхности около 30-50 же, а вот планета, насколько помню, очень разрежена. Она раздулась просто из-за перегрева, горячий юпитер. Весит наверно как два-три Юпитера. А звезда уже на 20 масс Юпи потянет. Для запуска ядерного синтеза нужно 18 масс
Короче, планета вращалась бы вокруг звёзды. Да и такая тусклая звезда не смогла бы поддерживать нагрев этого горячего юпитера, и он бы быстро остыл, сжавшись до более приемлемых размеров.
#507 #354089
>>354086
Rfrjq djghjc - nfrjq jndtn
#508 #354091
Читал это https://geektimes.ru/post/293187/ говно и что-то не понял.

>Постоянное ускорение требует постоянного увеличения тяги


Чо? Они ебанулись? В смысле, разве недостаточно иметь неменяющуюся тягу (и ускорение) и постепенно к скорости света приближаться?
#509 #354094
>>354091
Какую-то хуиту несут. Требуемая энергия для увеличения скорости на, допустим, 10 км/с, конечно будет постепенно расти, но процентов до 80 от скорости света этот рост будет почти не заметен.
И вообще, умные аноны, скорость роста количества необходимой энергии будет примерно расти так же как и коэффициент замедления времени, нет? Замедлилось время в два раза, значит и энергии в два раза больше надо, всегда так думал
#510 #354095
>>354094
Ну я всегда думал что только сопротивление среды влияет на кол-во энергии необходимое для поддержания ускорения. Вообще не понимаю что это за хуйня такая и о чем они там написали.
#511 #354096
>>354082

>во вселенной?


Охуел совсем? Мы в родной галактике звезды пересчитать очно не можем.
>>354083

>Кто


Можешь сам определить, при наличии соответствующих знаний и инструментов.

>как


По смещению спектра звезд.
Если они отдаляются, то спектр смещается в красную область, если приближаются, то в синюю.
#512 #354101
>>354091
С увеличением скорости масса тела тоже будет увеличиваться, ну а для разгона большей массы нужна большая энергия. И вообще просто читай ТО
#513 #354104
>>354101

>С увеличением скорости масса тела тоже будет увеличиваться


Вот я асёл, простите, то не читал.
Впрочем это незначительно для скоростей даже на 60%с, так что я пропущу твой последний совет, когда-нибудь потом.
#514 #354105
>>354081
Есть к чему стремится.
А ты так и сдохнешь максимум через 50 лет
#515 #354106
>>354085
А то что будет?
#516 #354107
Чем больше ускорение, тем больше перегрузки воздействующие на организм, верно? А можно ли как-то скомпенсировать эти перегрузки чтобы условный хуман мог выжить при перелёте на ебовой скорости?
#517 #354108
>>354107
До определённого (небольшого) предела.
Люди в космосе нинужны, слава роботам!
#518 #354122
>>354088

>А звезда уже на 20 масс Юпи потянет. Для запуска ядерного синтеза нужно 18 масс


Это тред тупых вопросов, а не ответов. Минимальная масса коричневого карлана порядка 70 масс юпитера, и там и то не будет толькового термояда.
#519 #354124
>>354122

>порядка 70 масс юпитера


Это красный уже будет, минимальный коричневый ~ 10 масс, но он тусклый будет очень.
Для стабильного наиболее тусклого красного вроде нужно не меньше 80 масс Юпитера.
#520 #354125
>>354124

>тусклый


Настолько тусклый, что верхние слои его атмосферы вполне могут иметь криогенные температуры, если его не будет ничего греть со стороны.
#521 #354127
>>354095
Насколько я понял, для тебя масса той же остаётся, просто время ускоряется и само ускорение замедляется для стороннего наблюдателя, а ты так и продолжаешь стабильно набирать скорость.
#522 #354131
>>353714
Тебя в гугле забанили?
https://ru.m.wikipedia.org/wiki/Лёд_VII
#523 #354132
>>354087

>почему его не распидорашивает


А почему балон с метаном в гараже не распидорашивает?
#524 #354141
>>354101
А как насчёт фотона? Шах и мат!
KeplerView.jpg1 Мб, 600x890
#525 #354146
Это правда, что Кеплер только в эту мизерную область смотрел и нашел всех своих кандидатов там? И звезду Табби там же? Он не вращался, не смотрел никуда в другие места?
#526 #354147
>>354146
Правда.
Там же.
Не смотрел.
Суть в том, что ему нужно "смотреть" на эти звезды продолжительное время, так как нужно засечь транзит планеты, а они могут иметь и долгие периоды обращения.
ТЕСС642 Кб, 1280x1024
#527 #354148
>>354147
Вот ты говоришь долго нужно смотреть, а хули они ТЕСС запускают, который не в одну точку смотрит, а делает обзор всего небо, постоянно смещая поле зрения? Вот на видео говорят
https://www.youtube.com/watch?v=UGvZ6-bM3pQ

Они же кучу планет пропустят, постоянно смещая камеру куда-то в новое место. Хотя даже так они обещают открыть немыслимые 30000 планет за 2 года работы.
#528 #354151
>>354148
TESS будет смотреть на более короткую дистанцию, на более яркие звезды, соответственно и светил в глубину будет меньше.
грубо говоря он возьмет числом, в смысле покрытием площади.

>Они же кучу планет пропустят


>2 года работы


За два года они в любом случае пропустят многие планеты, например, Марс имеет период более двух лет. То есть, если предположить, что подобным ТЕССом на нас смотрели бы воображаемые хренопланетяне, то они бы решили, что у нас 4 планеты в системе.
20170509-00010003-piaeigat-000-view189 Кб, 1280x1811
#529 #354152
30к экзопланет откроем сука, 30к экзопланет. И это с учетом кеплеровских кандидатов получится 33к экзопланет до 2020 годов, когда в ход начнут вступать экстремальные земные телескопы. Потом еще PLATO запустят, или может еще китайцы свой кеплер сделают. До 2030 года смогут открыть больше 100000 экзопланет. Я даже не представляю как это повлияет на астрономию и на человечество. Планетология станет самой популярной наукой наверное, среди 100к планет делать открытия можно направо и налево, находить экстремальные планеты рекордсмены в любой категории(самые маленькие, тяжелые, яркие, плотные, с самым большим магн полем, кольцами, атмосферой итд), открывать луны, кольца, кометы, выдвигать теории, пересматривать Солнечную систему, давать новым планетам названия, требовать деньги на новые телескопы, рисовать картинки, делать игры, писать книги, строить симуляции(Space Engine разбухнет), читать лекции, обсуждать все это говно.
#530 #354153
>>354151
Да, спасибо, теперь понял. Яркие звезды. Вот бы связки из нескольких ТЕСС спутников запустили и дОльше смотрели на предназначенные им участки неба. Хотя и так всё охуенно. Чем больше планет откроют, тем больше статистика, тем лучше можно выводы делать.
#531 #354154
>>354152
Нахуй ваши экзопланеты нинужны. Спейсач и так обсирается от всяких любителей алиенов, а тут лишний повод кукарекать А ВОТ НА ЭКЗАПЛАНЕТИНЕЙМ ЕСТЬ ЖИЗНЬ АНУС СТАВЛЮ.
#532 #354155
>>354154

>Нахуй ваши экзопланеты нинужны.


Долбоеба с двача забыли спросить, что нужно науке, а что не нужно.

>Спейсач и так обсирается


Пейсач переживет. У нас экзопланетный тред ведет один аутист и всем похуй.

>А ВОТ НА ЭКЗАПЛАНЕТИНЕЙМ ЕСТЬ ЖИЗНЬ АНУС СТАВЛЮ.


Кеплер нашел как минимум 25 экзопланет земной группы в зоне обитаемости и пейсач это пережил.
В первый раз вижу дегенерата в пейсаче, который против новых открытий.
#533 #354158
>>354141
Лалка, ну ты же ведь знаешь, что при перемещении через пространство фотон не имеет массы?
#534 #354159
>>354155
Мне на экзопланеты похуй тому шо мы никогда на них не попадем.
#535 #354161
>>354159
Хули ты тогда тут сидишь? Ты даже до Луны никогда не долетишь
#536 #354162
>>354161
На Луну за меня мурриканцы уже слетали.
#537 #354163
>>354162
Получается, тебе интересна только солнечная система? На все остальное в космосе тебе похуй?
#538 #354164
>>354155

> В первый раз вижу дегенерата в пейсаче, который против новых открытий.


ты новый просто совсем
#539 #354165
>>354163
Планеты похуй. Всякие йобы вроде Солнц, скоплений, туманностей и прочей годноты - не похуй.
#540 #354166
>>354165
Но до них ведь тоже не долететь, ты чо
#541 #354167
>>354166
Они хотя бы красивые.
#542 #354169
>>354167
Пиздец, а планеты как будто некрасивые, пейсэнджин накати
#543 #354172
>>354152

>Я даже не представляю как это повлияет ... на человечество.


Я представляю. Никак не повлияет.
#544 #354174
>>354169
Их только в пейсэнжине и можно рассматривать, тогда как всякие туманности можно и на йоба-фоточках насы, и в телескоп посмотреть.
#545 #354176
>>354174

> и в телескоп посмотреть


шо то мутное пятно, шо то
#546 #354180
>>354176
Мутные пятна в ирл телескоп >>>> сраный пиксель экзопланет на фоточках
#547 #354184
>>354159

экзопланеты интересны с научной точки зрения в плане понимания формирования и жизненного цикла планет и звездных систем, ну и всего остального что с этим связано, в том числе атмосфера, вода и всё такое. даже если мы никогда до них не долетим это поможет нам понять прошлое, настоящее и будущее нашей солнечной системы и её планет

спейсаны, а я вот низкоинтеллектуальный и чегото не понимаю, а транзитным методом получается можно найти только планеты у звезд у которых проекция плоскости эклиптики проходит через нашу планету?
#548 #354185
>>354184
Ага. Остальные по колебаниям родительских звезд находят, или, изредка, путем визуальных наблюдений
#549 #354188
>>354185
А плоскости эклиптики у звезд одной галактики примерно совпадают, или рандомно-ориентированные?
#550 #354189
>>354158
А импульс сука такая имеет
#551 #354190
>>354188
Тут я не авторитет, но вроде рандомно. По крайней мере эклиптика солнечной системы почти перпендикулярна эклиптике Млечного пути
#552 #354191
>>354190
Поэл.
#553 #354198
Вот все ищут предельную массу нейтронных звёзд. А может быть так, что она намного выше чем предполагается, за счёт того что все самые лёгкие ЧД образуются под давлением сверхновых?
#554 #354199
>>354198
Что сказал, епта?
1014166541335422 Кб, 703x181
#555 #354205
>>354184
>>354159
>>354172
>>354180
Вы все забыли тут про БУЛЬЕН с разрешением в 1 метр.
#556 #354207
>>354205

>Недельный пинг


Мило.
#557 #354209
>>354207
Три-четыре дня пинг. Да и не похуй ли какой пинг?
Зато посмотришь как нормальные люди живут. Вон, Жалфдыоаропжсл-то Аофыдвоварф уже звездолёт строит и на землю сигналы через лазер отправляет, а ты сидишь всё дома. И не стыдно?
#558 #354211
>>354205
Осталось только доставить что-то уровня Уэбба за шестьсот астрономических единиц, при чем не как Пионеров, пульнул и пусть летят, надо будет ещё в точке прибытия затормозить и выйти на орбиту вокруг Солнца! Ну или компенсировать постоянно гравитацию Солнца двигателями, если для наблюдения нужно в одной точке оставаться. Мало того,с этой йобы можно будет только на одну звёздную систему посматреть, чтобы сдвинуть точку зрения телескопа на 1 градус, нужно будет сам телескоп на 10 а.е. сдвинуть. Хуйня без задач короче. Пока долетит до окраин системы гипердрайв какой изобретут
#559 #354215
>>354209
Пинг -это туда и ответ оттуда. 547 а.е. - это неделя.
#560 #354227
>>354211

>надо будет ещё в точке прибытия затормозить


Тормозить необязательно, фокус и на 1к а.е. будет нормас.

>Ну или компенсировать постоянно гравитацию Солнца двигателями, если для наблюдения нужно в одной точке оставаться


Скорее придется учитывать орбиту планеты и вставать на такую солнечную орбиту, при которой наблюдать можно больше одного часа в тысячу лет. Хотя тебе и одного часа будет достаточно чтобы получить тысячи терабайт данных. Но вообще это конечно пизда как сложно.

>Мало того,с этой йобы можно будет только на одну звёздную систему посматреть, чтобы сдвинуть точку зрения телескопа на 1 градус, нужно будет сам телескоп на 10 а.е. сдвинуть.


А зачем его двигать? Если орбита будет норм, можно всегда и наблюдать, норм будет. Можно даже смотреть как там местные животные (если есть) стайками ходят.

Теоретически с помощью такого телескопа можно даже поймать сигнал лазера какого-нибудь еблана, который уже вычилил как, куда и когда светить чтобы мы (вернее жители этой планетки) поймали этот сигнал.
Лазер достаточной мощности может позволить себе каждый, останется только сделать простейший спутник который будет работать 10к лет и передавать сообщения вида этот чел >>354211 еблан, азаз, первый контакт хуй лососайте
И почему-то я уверен что не первый подумал про такой способ передачи информации как 100% осуществимый на любых технологических стадиях развития цивилизаций.
В удивительный ты век живешь, товарисч, не пизди про "беззадач".

>>354215
Ануда, я тупой. Вопщем-то телескоп такой полностью автоматический будет, хуй знает зачем на него что-то отправлять.
#561 #354230
>>354227

>Вопщем-то телескоп такой полностью автоматический будет, хуй знает зачем на него что-то отправлять.


А ты не очень умный, да?
#562 #354231
>>354230

>Ануда, я тупой.


>А ты не очень умный, да?


Ты жопой читаешь?
И если ты возражаешь - возражай не как быдло или ребенок с 8б, а ставь конкретные вопросы или конкретные утверждения.
#563 #354236
>>354231
Этот телескоп ещё и нам что-то присылать должен. А с Земли команды подавать ему
#564 #354237
>>354236
Какие команды подавать, фантазер? У тебя и орбиты и действия просчитаны, считай, лет за 10 до фактической отправки телескопа. Я хз что там по-твоему требует ручной (?) настройки на расстоянии.
#565 #354239
>>354237
Сомневаюсь что можно просчитать орбиту экзопланеты с достаточной точностью, чтобы телескоп смог навестить на нее через пятьдесят лет, минимум, за который телескоп преодолен такое расстояние
#566 #354240
>>354239

> Сомневаюсь


Не сомневайся.
#567 #354241
>>354237
Залетный не знает, что с АМС и телескопами работают десятки людей ежедневно.
tuXWeznskDA66 Кб, 576x567
#568 #354256
Короче, на Земле из-за особенностей наклона, полярный области по пол-года во тьме и по пол-года на свету. А какой ширины (сколько десятков или даже сотен километров) область, по обе стороны от границы которой чётко видна разница, где ЕЩЁ сутки сменяются полноценной, ежедневно сменяя день ночью, а где УЖЕ день длится половину года, как и ночь?
Если можно, хотелось бы получить ответ на примере каких-нибудь мест в России, а ещё лучше на примере конкретных насёлённых пунктов.

Из этого вопроса вытекает второй, более научный: а есть ли какая-нибудь формула, позволяющая рассчитывать ширину такой сумеречной зоны, с учётом размеров планеты и звезды, вокруг которой вращается планета, с учётом наклона оси, скорости вращения вокруг своей и вокруг звёздной оси и т.д.?
#569 #354273
>>354241
Ты вообще понимаешь разницу между ними и назначение такого телескопа, а? Портфель собери, олдфак раздела.
wordpress-d44b91e4-58c2-4cc7-97f8-30e9535a196864 Кб, 347x560
#570 #354310
>>354256

>чётко


Нет такой границы. Гугли гражданские сумерки, а затем и все остальные. Там, где проходит условная линия, технически разделяющая полярный день и белые ночи, разницы не увидишь. Потому что технически разница заключается в том, что если краешек Солнца над горизонтом, то это день. А если Солнце уже чуть за горизонтом - то это гражданские сумерки. Наличие этих сумерек подразумевает понятие белых ночей, которые вообще мало чем отличаются от полярного дня по яркости.
Работал пару лет назад в республике Коми и прихуел порядком, когда мы с одним прорабом, приехав в локацию 64-го градуса северной широты в половину второго ночи, увидели освещенность уровня ясного летнего утра московских широт. Светло было, будто часов 8 утра.
Сейчас же работаю на Ямале, 71-й градус с. ш. Полярный день длился 80 суток с копейками, закончился 2-го штоле августа, но я так ни разу и не вышел посреди ночи посмотреть на Солнце над горизонтом, потому что блять тучи очень часто висят в небе и нихуя звездочку не видно. Вот тебе и полярный день. Хотя лето вполне ясным и теплым было по местным меркам. Олсо

>от границы, где ЕЩЁ сутки сменяются полноценной, ежедневно сменяя день ночью, а где УЖЕ день длится половину года, как и ночь?


Такой границы в принципе не может существовать. По полгода день и ночь длятся на полюсах строго, да и то из-за особенностей физики этого мира, а именно рассеивания света в атмосфере, полярная ночь таки чуть короче полугода. Чем дальше от полюсов к экватору уходят широты, тем короче длительность полярных дня и ночи. В вышеупомянутой локации на Ямале, в Сабетте, цикл соответствует приблизительно 80-ти дням, и день, вроде как, таки на пару суток длиннее.
#571 #354314
>>354273
Знаю, что ты дурачок, который нихуя не знает, но дохуя пиздит.
#572 #354316
>>354256

>а есть ли какая-нибудь формула, позволяющая рассчитывать ширину такой сумеречной зоны


Есть Санкальк
http://suncalc.net
#573 #354319
>>354314
Портфель соберай, олдфак.
#574 #354323
>>354319
Мне завтра ко второму.
#575 #354327
>>354319

>соберай


Мммм.
#576 #354329
Можно как-то мониторить регистрируемые сверхновые? Такая-то вспышка оттуда-то за таким-то временем, например.
15024329064980120 Кб, 800x549
#578 #354350
возьмут ли phd с судимостью в комонавы?
#579 #354371
>>354349
Не знаю кто такой пхд, но нет. Тут вон один пытался https://geektimes.ru/post/288802/
Тред утонул или удален.
Это копия, сохраненная 24 декабря 2017 года.

Скачать тред: только с превью, с превью и прикрепленными файлами.
Второй вариант может долго скачиваться. Файлы будут только в живых или недавно утонувших тредах. Подробнее

Если вам полезен архив М.Двача, пожертвуйте на оплату сервера.
« /spc/В начало тредаВеб-версияНастройки
/a//b//mu//s//vg/Все доски